Крок 1 - Медицина 2002 (буклет)

1 / 200
Через деякий час після інтенсивної фізичної тренування у спортсмена активується глюконеогенез, основним субстратом якого є: After some time after intense physical training, the athlete activates gluconeogenesis, the main substrate of which is:

Серин Serine

Глутаминовая кислота Glutamic acid

Лактат Lactate

альфа - кетоглутарат alpha-ketoglutarate

Аспарагінова кислота Aspartic acid

2 / 200
На місці автокатастрофи пораненому водію наклали кровоспинний джгут на верхню третину стегна. У хірургічне відділення хворий був доставлений в задовільному стані тільки через 3 години. Після зняття джгута з'явився різкий набряк тканини стегна, частий пульс, холодний піт, різка гіпотонія . Яке патологічне явище розвинулося у потерпілого? At the scene of the car accident, a hemostatic tourniquet was applied to the upper third of the thigh of the injured driver. The patient was brought to the surgical department in a satisfactory condition only 3 hours later. After the tourniquet was removed, a sharp swelling of thigh tissue, frequent pulse, cold sweat, sharp hypotension. What pathological phenomenon developed in the victim?

Кардіогенний шок Cardiogenic shock

Токсеміческій шок Toxemic shock

Геморагічний шок Hemorrhagic shock

Анафілактичний шок Anaphylactic shock

Колапс Collapse

3 / 200
Хворому з значними опіками зробили пересадку донорської шкіри, але на 8-му добу трансплантат набряк, змінився його колір і на 11-ту добу він почав відторгатися. Які клітини беруть в цьому участь? A donor skin transplant was performed on a patient with significant burns, but on the 8th day the graft swelled, changed its color, and on the 11th day it began to reject. What cells participate in this?

Базофіли Basophils

Т - лімфоцити T - lymphocytes

Еритроцити Erythrocytes

Еозинофіли Eosinophils

В - лімфоцити B - lymphocytes

4 / 200
Хворому для купірування кишкової коліки призначений атропіну сульфат. Яке з названих захворювань може служити протипоказанням в цьому випадку? The patient is prescribed atropine sulfate to relieve intestinal colic. Which of the named diseases can serve as a contraindication in this case?

Глаукома Glaucoma

Бронхіальна астма Bronchial asthma

Запаморочення Dizziness

Синусова брадикардія Sinus bradycardia

Гіпотонія Hypotonia

5 / 200
Біля інфікованої рани збільшилися регіонарні лімфовузли. При гістологічному дослідженні в них виявлено збільшення кількості макрофагів, лімфоцитів і лімфатичних фолікулів в кірковому шарі, а також велику кількість плазматичних клітин. Який процес в лімфатичних вузлах відображають виявлені гістологічні зміни? Regional lymph nodes increased near the infected wound. Histological examination revealed an increase in the number of macrophages, lymphocytes, and lymphatic follicles in the cortical layer, as well as a large number of plasma cells. What process in the lymph nodes reflect the detected histological changes?

Реакцію гіперчутливості Hypersensitivity reaction

Пухлинну трансформацію Tumor transformation

Придбану недостатність лімфоїдної тканини Acquired insufficiency of lymphoid tissue

Антигенну стимуляцію Antigenic stimulation

Уроджену недостатність лімфоїдної тканини Congenital insufficiency of lymphoid tissue

6 / 200
У дитини підвищилася температура тіла, з'явилися болі при ковтанні. При огляді: піднебінні мигдалини збільшені, темно - червоні, покриті сіро - жовтими плівками, щільно спаяні з поверхнею мигдаликів. Яке запалення розвинулося в мигдалинах? The child's body temperature increased, pains appeared when swallowing. On examination: the palatine tonsils are enlarged, dark red, covered with gray-yellow films, tightly soldered with surface of the tonsils. What kind of inflammation developed in the tonsils?

Геморагічне Hemorrhagic

Крупозне True

Катаральне Catarrhal

Дифтеритичне Diphtheritic

Гнійне Purulent

7 / 200
У пацієнта, які тривалий час хворіє на туберкульоз, виявлено внутрішньоклітинне розташування мікобактерій. Який з вказаних препаратів обов'язково повинен бути включений в комплексну терапію туберкульозу? The intracellular location of mycobacteria was detected in a patient who has been suffering from tuberculosis for a long time. Which of the indicated drugs must be included in the complex therapy of tuberculosis?

Ізоніазид Isoniazid

Етамбутол Etambutol

Рифампіцин Rifampicin

Етіонамід Ethionamide

Натрію парааміносаліцілат Sodium paraaminosalicylate

8 / 200
Після прийому жирної їжі у хворого з'являються нудота і печія, має місце стеаторея. Причиною такого стану може з'явитися: After eating fatty food, the patient develops nausea and heartburn, steatorrhea occurs. The cause of this condition may be:

Порушення синтезу трипсину Trypsin synthesis disorder

Підвищений виділення ліпази Increased release of lipase

Брак амілази Amylase deficiency

Порушення синтезу фосфоліпази Disruption of phospholipase synthesis

Брак жовчних кислот Lack of bile acids

9 / 200
Під час здачі іспиту у студентів ' пересихає в роті '. Посилена реалізація яких рефлексів є механізмом, що обумовлює розвиток цього стану? During the exam, students have a 'dry mouth'. Enhanced implementation of which reflexes is the mechanism that causes the development of this condition?

Незаперечних периферичних Undisputed Peripherals

умовних парасимпатических conditional parasympathetic

Незаперечних парасимпатических Undeniable parasympathetic

Незаперечних симпатичних Undeniably cute

Умовних симпатичних Conditionally cute

10 / 200
Хворому з цирозом печінки разом з лікарськими препаратами внутрішньовенно ввели 500,0 мл 5% розчину глюкози. Яке порушення водного балансу найбільш ймовірно може виникнути у хворого? 500.0 ml of a 5% glucose solution was administered intravenously to a patient with cirrhosis of the liver along with medications. What water balance violation is most likely to occur in the patient?

Порушень водного балансу НЕ відбудеться The water balance will NOT be violated

Гиперосмолярная гипергидратация Hyperosmolar hyperhydration

Гіпоосмолярна гипергидратация Hyposmolar hyperhydration

Гіпоосмолярна дегідратація Hyposmolar dehydration

Ізоосмолярна гипергидратация Isoosmolar hyperhydration

11 / 200
У гістологічному препараті визначається ться рецепторная зона сенсоепітеліального органу почуттів. Клітини даної зони лежать на базальної мембрані і включають такі види: зовнішні і внутрішні сенсорні, зовнішні і внутрішні фалангові, стовпові, зовнішні прикордонні і зовнішні підтримують. Якому органу почуттів належить дана рецепторная зона? The receptor zone of the sensorepithelial sense organ is determined in the histological preparation. The cells of this zone lie on the basement membrane and include the following types: external and internal sensory, external and internal phalangeal, columnar, outer border and outer supporting. What sense organ does this receptor zone belong to?

Органу рівноваги Equilibrium body

Органу зору Organ of vision

Органу слуху The organ of hearing

Органу нюху Organ of smell

Органу смаку Taste organ

12 / 200
У чоловіка при ураженні одного з відділів ЦНС спостерігається астенія, м'язова дистонія, порушення рівноваги. Який з відділів ЦНС вражений? A man has asthenia, muscle dystonia, and imbalance when one of the CNS departments is affected. Which CNS department is affected?

Чорна субстанція Black substance

Мозочок Cerebellum

Червоні ядра Red kernels

Вестибулярні ядра Vestibular nuclei

Ретикулярная формація Reticular formation

13 / 200
Після введення пирога у хворого підвищилася температура, шкірні покриви стали блідими, холодними на дотик, з'явився озноб, зрілось споживання кисню. Як змінюються процеси терморегуляції в описаному періоді лихоманки? After the introduction of the pie, the patient's temperature rose, the skin became pale, cold to the touch, chills appeared, and oxygen consumption increased. How do thermoregulation processes change in the described period fever?

Збільшується тепловіддача The heat transfer is increasing

Знижується теплопродукція Decreasing heat production

Тепловіддача дорівнює теплопродукції Heat output is equal to heat production

Збільшується теплопродукція Increasing heat production

Знижується тепловіддача Decreasing heat output

14 / 200
У травмпункт звернувся чоловік 45 років після побутової травми плеча. Об'єктивно: відсутні функції розгинання, приведення і пронації плеча. Пошкодження який м'язи викликало таке стан? A 45-year-old man came to the trauma center after a domestic shoulder injury. Objectively: the functions of shoulder extension, adduction, and pronation are absent. What muscle damage caused this condition?

малий круглий м'яз small round muscle

підлопатковий м'яз subscapular muscle

великий круглий м'яз large round muscle

надостьовий м'яз suprastus muscle

підостьовий м'яз podium muscle

15 / 200
У препараті червоного кісткового мозку людини визначаються скупчення гігантських клітин, що знаходяться в тісному контакті з синусоїдними капілярами. Які формені елементи крові утворюються з цих клітин? In the preparation of red bone marrow of a person, clusters of giant cells are determined, which are in close contact with sinusoidal capillaries. What formed blood elements are formed from these cells?

Еритроцити Erythrocytes

Моноцити Monocytes

Лейкоцити Leukocytes

Лімфоцити Lymphocytes

Кров'яні пластинки Platelets

16 / 200
При гістологічному дослідженні стулки мітрального клапана серця виявлено: мукоїдному набряк, пошкодження ендотелію, утворення по замикаючому краю фібринових тромбів. Яка форма ревматичного ендокардиту спостерігається? During a histological examination of the heart mitral valve leaf, the following was revealed: mucoid edema, damage to the endothelium, formation of fibrin thrombi at the closing edge. What form of rheumatic endocarditis is observed?

Фібропластичний ендокардит Fibroplastic endocarditis

Поліпозно - виразковий ендокардит Polyposis - ulcerative endocarditis

Дифузний ендокардит Diffuse endocarditis

Гострий бородавчастий ендокардит Acute warty endocarditis

Зворотно - бородавчастий ендокардит Reverse - warty endocarditis

17 / 200
У дівчинки 5 років спостерігається висока температура і біль в горлі. Об'єктивно: набряк м'якого піднебіння, на мигдалинах сірі плівки, які відділяються з працею, залишаючи глибокі кровоточать, дефекти тканини. Яке з нижчеперелічених захворювань найбільш ймовірно? A 5-year-old girl has a high temperature and a sore throat. Objectively: swelling of the soft palate, gray films on the tonsils, which separate with difficulty, leaving deep bleeding, tissue defects. Which of the following diseases is most likely?

Лакунарна ангіна Lacunar angina

Інфекційний мононуклеоз Infectious mononucleosis

Некротична ангіна Necrotic sore throat

Дифтерія зіва Pharyngeal diphtheria

Ангіна Симон івського - Венсана Symon ivsky's Angina - Vincent

18 / 200
У хворого 70 років діагностовано крововилив у стовбур мозку. Обстеження виявило підвищення тонусу м'язів - згиначів на тлі зниження тонусу мишцразгібателей. Роздратуванням яких структур мозку можна пояснити зміни в тонусі м'язів? A 70-year-old patient was diagnosed with a brain stem hemorrhage. The examination revealed an increase in the tone of the flexor muscles against the background of a decrease in the tone of the extensors. The irritation of which brain structures can explain the changes in muscle tone?

ядер Дейтерса Deiters core

Чорної субстанції Black substance

Червоних ядер Red cores

ядер Голя Holy core

ядер Бурдаха Burdach kernel

19 / 200
У хворого спостерігається атонія м'язів. Назвіть фермент м'язової тканини, активність якого може бути знижена при такому стані. The patient has muscle atony. Name the enzyme of the muscle tissue, the activity of which can be reduced in this condition.

Креатинфосфокиназа Creatine phosphokinase

Амілаза Amylase

транскетолази transketolases

Каталаза Catalase

7- глутамінтрансферази 7- glutamine transferase

20 / 200
У хворого виявлено різке зниження активності сурфактанту легень. Які зміни слід очікувати у цього хворого? The patient has a sharp decrease in lung surfactant activity. What changes should be expected in this patient?

Зменшення трахеобронхіального секрету Decreased tracheobronchial secretion

Зміна еластичних властивостей легенів Change in the elastic properties of the lungs

Розростання сполучної тканини легенів Proliferation of connective tissue of the lungs

Порушення кровообігу в легенях Disruption of blood circulation in the lungs

Схильність альвеол до спадання і неможливість їх швидкого розправлення Propensity of alveoli to collapse and the impossibility of their quick straightening

21 / 200
52- річну пацієнтку на протязі останніх декількох днів турбують напади болю в правому підребер'ї після прийому жирної їжі. Візуально визначається пожовтіння склер і шкіри, ахолічний кал, сеча ' кольору пива? Присутність якого речовини в сечі пацієнтки зумовило темний колір сечі при обтураційній жовтяниці? A 52-year-old female patient has been bothered by pain attacks in the right hypochondrium for the past few days after eating fatty food. Yellowing of the sclera and skin, acholic stool, urine is visually determined 'the color of beer? The presence of which substance in the patient's urine caused the dark color of urine in obturating jaundice?

стеркобіліна sterkobilina

Кетонових тел Ketone bodies

білірубінглюкуроніди bilirubin glucuronides

уробилин Urobilin

Глюкози Glucose

22 / 200
На обмеженому ділянці епідермісу внаслідок травми відсутні шари аж до росткового. Які клітини послужать джерелом його регенерації? On a limited area of the epidermis due to trauma, there are no layers up to the germ layer. What cells will serve as a source of its regeneration?

Шар базальних клітин незруйнованим ділянки Layer of basal cells of the intact area

Шари базальних клітин Basal cell layers

Шари крилатих клітин Layers of winged cells

Шари крилатих і базальних клітин Layers of winged and basal cells

Шари крилатих і зернистих клітин незруйнованим ділянки Layers of winged and granular cells of the intact area

23 / 200
У хворого спостерігається сльозотеча, підвищене слиновиділення. Це стан, спільно з іншими симптомами, расц енівается як роздратування волокон одного з черепних нервів. Якого саме нерва і яких його волокон? - Соматичні рухові волокна лицьового нерва The patient has lacrimation, increased salivation. This condition, together with other symptoms, is considered as irritation of the fibers of one of the cranial nerves. Which nerve and which fibers ? - Somatic motor fibers of the facial nerve

Парасимпатические волокна блукаючого нерва Parasympathetic fibers of the vagus nerve

Соматичні рухові волокна окорухового нерва Somatic motor fibers of the oculomotor nerve

Парасимпатические волокна лицьового нерва Parasympathetic fibers of the facial nerve

Парасимпатические волокна окорухового нерва Parasympathetic fibers of the oculomotor nerve

24 / 200
У хворого з Тімом (пухлина вилочкової залози) спостерігається ціаноз, розширення підшкірної венозної сітки і набряк м'яких тканин обличчя, шиї, верхньої половини тулуба і верхніх кінцівок. Який венозний стовбур перетиснено пухлиною? A patient with Tim (tumor of the thymus gland) has cyanosis, expansion of the subcutaneous venous network, and swelling of the soft tissues of the face, neck, upper half of the body, and upper limbs. Which venous trunk is compressed by the tumor?

Внутрішня яремна вена Internal jugular vein

Передня яремна вена Anterior jugular vein

Підключичної вена Subclavian vein

Верхня порожниста вена Superior vena cava

Зовнішня яремна вена External jugular vein

25 / 200
Під час розтину в верхній частці правого легкого про назовні великий клиновидний вогнище темно - червоною щільною тканини. Під час гістологічного дослідження в ній виявлений некроз стінок альвеол ; просвіт альвеол щільно заповнений еритроцитами. Який процес розвинувся в легенях? During the autopsy in the upper lobe of the right lung, a large wedge-shaped focus of dark red dense tissue was found. During histological examination, necrosis of the alveolar walls was found in it; the lumen of the alveoli was dense filled with erythrocytes. What process developed in the lungs?

Ателектаз легенів Atelectasis of the lungs

Гангрена легенів Gangrene of the lungs

Геморагічний інфаркт легень Hemorrhagic pulmonary infarction

Крововилив в легені Hemorrhage in the lungs

карніфікація легких carnification of lungs

26 / 200
При яких групах крові батьків по системі резус можлива резус - конфліктна ситуація під час вагітності? With which blood groups of the parents according to the Rhesus system, Rhesus is possible - a conflict situation during pregnancy?

Дружина Rh +, чоловік Rh + (гетерозигота) Wife Rh +, husband Rh + (heterozygous)

Дружина Rh +, чоловік Rh + (гомозигота) Wife Rh +, husband Rh + (homozygous)

Дружина Rh-, чоловік Rh + (гомозигота) Wife Rh-, husband Rh + (homozygous)

Дружина Rh-, чоловік Rh- Wife Rh-, husband Rh-

Дружина Rh + (гетерозигота), чоловік Rh + (гомозигота) Wife Rh + (heterozygous), husband Rh + (homozygous)

27 / 200
Мукополісахарідоз відноситься до хвороб накопичення. З - за відсутності ферментів порушується розщеплення полісахаридів. У хворих спостерігається вище ня виділення їх з сечею і накопичення в одному з органоїдів клітин. В яких органелах накопичуються мукополісахариди? Mucopolysaccharidosis refers to diseases of accumulation. C - in the absence of enzymes, the splitting of polysaccharides is disturbed. In patients, there is an increase in their excretion in the urine and their accumulation in one of the cell organoids. In which organelles accumulate mucopolysaccharides?

У клітинному центрі In the cell center

В ендоплазматичному ретикулуме In the endoplasmic reticulum

В лізосомах In lysosomes

У комплексі Гольджі In the Golgi complex

У мітохондріях In mitochondria

28 / 200
З метою аналгезії можуть бути використані речовини, що імітують ефекти морфіну, але виробляються в ЦНС. Вкажіть таке речовина. For the purpose of analgesia, substances that mimic the effects of morphine, but are produced in the central nervous system, can be used. Specify this substance.

Окситоцин Oxytocin

бетаендорфін betaendorphin

Кальцитонин Calcitonin

Вазопрессин Vasopressin

Соматоліберин Somatoliberin

29 / 200
У хворого, які тривалий час кращого тютюн, розвинувся рак легені. Які з перерахованих канцерогенних речовин містяться в тютюновому димі і відносяться до ПАУ (поліненасичених ароматичним вуглеводів)? A patient who has been using tobacco for a long time developed lung cancer. Which of the listed carcinogenic substances are contained in tobacco smoke and belong to PAHs (polyunsaturated aromatic carbohydrates)?

бета - нафтиламин beta - naphthylamine

диметиламіноазобензолу dimethylaminoazobenzene

Ортоаміноазотолуол Orthoaminoazotoluene

Бензпирен Benzpyrene

Діетілнітрозамін Diethylnitrosamine

30 / 200
У померлого 58 років на розтині мітральний клапан деформований, потовщений, змикається НЕ до кінця. Мікроскопічно: вогнища колагенових волоконец еозинофільно, дають плюсову реакцію на фібрин. Найбільш ймовірно це: At the autopsy of a deceased 58-year-old, the mitral valve is deformed, thickened, does not close completely. Microscopically: foci of collagen fibers are eosinophilic, give a positive reaction to fibrin. This is most likely :

Гіаліноз Hyalinosis

Мукоїдне набухання Mucoid swelling

Фібринозне запалений ие Fibrinous inflammatory disease

Амилоидоз Amyloidosis

Фібриноїдне набухання Fibrinoid swelling

31 / 200
Хворий 45 років госпіталізований в хірургічне відділення з скаргами на раптову гостру біль в надчеревній області. Після обстеження поставлений діагноз: перфоративная (проривна) виразка задньої стінки шлунка. Куди вилилося вміст шлунка в момент перфорації? A 45-year-old patient was hospitalized in the surgical department with complaints of sudden sharp pain in the epigastric region. After the examination, a diagnosis was made: a perforating (breakthrough) ulcer of the back wall of the stomach. Where did it spread stomach contents at the time of perforation?

В печінкову сумку In the liver bag

В лівий брижових синус In the left mesenteric sinus

У правий брижових синус In the right mesenteric sinus

В сальниковую сумку In the stuffing bag

У передшлункову сумку In the pregastric bag

32 / 200
При попаданні чужорідного тіла в дихальні шляхи на який бронх має бути направлено увагу лікаря в першу чергу для пошуку стороннього тіла і його видалення? When a foreign body enters the respiratory tract, which bronchus should the doctor's attention be directed to in the first place in order to find the foreign body and remove it?

Ліві сегментарні бронхи Left segmental bronchi

Лівий головний бронх Left main bronchus

Правий головний бронх Right main bronchus

Ліві часткові бронхи Left partial bronchi

Праві часткові бронхи Right partial bronchi

33 / 200
Смерть хворого 16 років настала від розлитого (поширеного) фібрінозногнойного перитоніту. На розтині в нижньому ділянці тонкої кишки виявлена виразка, що повторює форму пеєрових бляшки, з перфорацією стінки кишки. Мікроскопічна дослідження виявило невизначеність малюнка лімфоїдної тканини, вите снення її проліферуючими моноцитами, які формують гранульоми. Ускладнення якого захворювання стало причиною смерті? The death of a 16-year-old patient was caused by disseminated (diffuse) fibrinous-purulent peritonitis. An autopsy revealed an ulcer in the lower part of the small intestine, repeating the shape of Peyer's plaques, with perforation of the intestinal wall . Microscopic examination revealed the indeterminacy of the pattern of lymphoid tissue, its destruction by proliferating monocytes that form granulomas. What complication of the disease caused death?

черевного тифу typhoid

бруцельоз brucellosis

дизентерія dysentery

холери cholera

неспецифічні виразкового коліту nonspecific ulcerative colitis

34 / 200
Після перенесеного запального захворювання у хворого виникло неповне відведення очного яблука в латеральну сторону. Який нерв пошкоджений у хворого? After suffering an inflammatory disease, the patient developed an incomplete deviation of the eyeball to the lateral side. What nerve is damaged in the patient?

Окоруховий Щculomotor

Бічний Lateral

Особовий Personal

Зоровий Visual

Відвідний Abducent

35 / 200
У чоловіка 52 років діагностовано системний амебіаз з ураженням кишечника, печінки, легенів. Який препарат слід призначити? A 52-year-old man was diagnosed with systemic amoebiasis with damage to the intestines, liver, and lungs. What drug should be prescribed?

Хиниофон Kineophone

Метронідазол Metronidazole

Хінгамін Hingamine

Ентеросептол Enteroseptol

Тетрациклін Tetracycline

36 / 200
У крові хворого виявлено низький рівень альбумінів і фібриногену. Зниження активності яких органел гепатоцитів печінки наиб олее ймовірно обумовлює це явище? A low level of albumin and fibrinogen was detected in the patient's blood. A decrease in the activity of which organelles of the liver's hepatocytes most likely causes this phenomenon?

Лізосом Lysosome

Комплексу Гольджі Golgi complex

гранулярной ендоплазматичної мережі granular endoplasmic reticulum

Мітохондрій Mitochondrion

Гранулярності ендоплазматичної мережі Granularities of endoplasmic reticulum

37 / 200
У хворого виявлено множинні синці на тілі, тривалість кровотечі по Дуке 25 хвилин, число тромбоцитів крові 25 * 109 / л. Для якого захворювання характерні такі ознаки? The patient has multiple bruises on his body, Duque's duration of bleeding is 25 minutes, the number of blood platelets is 25 * 109 / l. What disease is characterized by such symptoms?

Авітаміноз С Avitaminosis C

Гемофілія В Hemophilia B

Гемофілія А Hemophilia A

Хвороба Віллебранда Willebrand's disease

Спадковий дефект освіти тромбоцитів Hereditary defect of platelet formation

38 / 200
У хворого жовтяницею отримані такі дані лабораторного обстеження: HbsAg-, HBeAs-, анти HBsG +, анти HBsM-, HCAg +. Який діагноз підтверджується на основі цих даних? A patient with jaundice received the following laboratory examination data: HbsAg-, HBeAs-, anti HBsG +, anti HBsM-, HCAg +. Which diagnosis is confirmed on the basis of these data ?

П овторное зараження гепатитом В Hepatitis B re-infection

Гепатит С Hepatitis C

Гепатит С, в анамнезі - гепатит Hepatitis C, history of hepatitis

Рецидив гепатиту В Hepatitis B recurrence

Хронічний гепатит В з низькою реплікативної активністю Chronic hepatitis B with low replicative activity

39 / 200
Хворому з бактеріальною інфекцією лікар призначив сульфадимезин в таблетках, порадивши запивати його 1,5-2 л лужної мінеральної води щодоби. Чим обумовлена необхідність даної рекомендації? The doctor prescribed sulfadimezin in tablets to a patient with a bacterial infection, advising him to drink 1.5-2 liters of alkaline mineral water with it. What is the need for this recommendation?

Для профілактики кристалізації ацетильних похідних препарату в ниркових канальцях For prevention of crystallization of acetyl derivatives of the drug in renal tubules

Для нейтралізації кислоти шлункового соку To neutralize stomach acid

Для зниження дратівної впливу на шлунок To reduce stomach irritation

Для пролонгування дії To extend the action

Для зсуву рН крові в лужну сторону To shift the blood pH to the alkaline side

40 / 200
У хворого 30 років з дизентерією, підтвердженої бактеріологічно, з'явилися ознаки парапроктиту. Про який стадії місцевих змін найбільш ймовірно йде мова у даного хворого? A 30-year-old patient with bacteriologically confirmed dysentery developed signs of paraproctitis. What stage of local changes is this patient most likely talking about?

Фолікулярний коліт Follicular colitis

Стадія загоєння виразок Stage of ulcer healing

Катаральний коліт Catarrhal colitis

Стадія освіти виразок Stage of formation of ulcers

Фіб рінозний коліт Fibrinous Colitis

41 / 200
У клінічно здорових людей в умовах високогір'я виявляються ознаки анемії. При аналізі крові виявляють серповидні клітини. Визначте генотип цих людей: Clinically healthy people in the highlands show signs of anemia. Blood tests reveal sickle cells. Determine the genotype of these people:

Aa Aa

аа aa

ХсХс XxXx

ХСХс ХСХс

АА AA

42 / 200
При визначенні групи крові по системі АВ 0 агглютинацию еритроцитів досліджуваної крові викликали стандартні сироватки Г і II груп і НЕ викликала сироватка III групи. Які агглютіногени містяться в цих еритроцитах? When determining the blood group according to the AB 0 system, agglutination of erythrocytes of the examined blood was caused by standard serums of groups G and II and not by serum of group III. What agglutinogens are contained in these erythrocytes?

С C

D і С D and C

А A

А і В A and B

В In

43 / 200
З випорожнень хворого дитини 6 місячного віку, який знаходився на штучному вигодовуванні, виділена культура кишкової палички з антигенною структурою 0-111. Який діагноз можна поставити? Escherichia coli culture with antigenic structure 0-111 was isolated from the feces of a sick 6-month-old child who was on artificial feeding. What diagnosis can be made?

Гастроентерит Gastroenteritis

Харчове отруєння Food poisoning

Коли - ентерит When - enteritis

дізентеріеподобние захворювання dysentery-like diseases

холероподібний захворювання cholera-like disease

44 / 200
Малюк попросив Вас надути повітряний кульку як можна сильніше за один видих. Яким обсягом повітря Ви скористаєтеся? The kid asked you to blow up the balloon as hard as you can in one exhalation. How much air will you use?

Резервним об'ємом вдиху Inspiratory reserve volume

Ємністю вдиху Inhalation capacity

Функціональної залишкової ємністю Functional residual capacity

Життєвої ємністю легких Vital capacity of the lungs

Загальною ємністю легких Total lung capacity

45 / 200
Під час хірургічної операції пацієнту проведено переливання крові. На антигени якого збудника необхідно перевірити цю кров? During a surgical operation, the patient received a blood transfusion. What antigens should this blood be tested for?

аденовіруси adenoviruses

Вірусу гепатиту В Hepatitis B virus

Вірусу гепатиту А Hepatitis A virus

ентеровірус enterovirus

Вірусу гепатиту Е Hepatitis E virus

46 / 200
У 6 місячного дитини спостерігалися часті і сильні підшкірні кровотечі. Призначення синтетичного аналога вітаміну К (вікасолу) дало позитивний ефект. У гамма - карбоксилювання глутамінової кислоти якого білка системи згортання крові приймає участь цей вітамін? Frequent and severe subcutaneous bleeding was observed in a 6-month-old child. The appointment of a synthetic analogue of vitamin K (Vikasol) had a positive effect. In gamma - carboxylation of glutamic acid of which protein of the coagulation system does this vitamin participate in the blood?

Фактора Розенталя Rosenthal factor

антигемофільний глобуліну А antihemophilic globulin A

Фактора Хагемана Hageman factor

Фібриногену Fibrinogen

протромбіну prothrombin

47 / 200
Хворому з гіперсекрецією шлункового соку лікар рекомендував виключити з дієти насичені бульйони і овочеві відвари, тому що вони стимулюють шлункову секрецію. Який переважний механізм стимуляції шлункової секреції в даному випадку? A patient with hypersecretion of gastric juice was advised by the doctor to exclude rich broths and vegetable broths from the diet because they stimulate gastric secretion. What is the predominant mechanism of gastric secretion stimulation in this case?'

Стимуляція вироблення секретину в 12-палої кишці Stimulation of secretin production in the duodenum

Роздратування механорецепторів порожнини рота Irritation of oral mechanoreceptors

Роздратування смакових рецепторів Taste buds irritation

Стимуляція вироблення гастрит ина G-клітинами Stimulation of production of gastritis by G-cells

Роздратування механорецепторів шлунка Irritation of stomach mechanoreceptors

48 / 200
Яка частина товстої кишки найбільш ймовірно буде пошкоджена при пораненні в праву половину живота? Which part of the colon is most likely to be damaged by a stab wound to the right half of the abdomen?

Сигмовидная ободова кишка Sigmoid colon

Пряма кишка Rectum

Поперечна ободова кишка Transverse colon

Висхідна ободова кишка Ascending colon

Низхідна ободова кишка Descending colon

49 / 200
Дитина 7 років НЕ може відвести плече і підняти його до горизонтального рівня, до особі руку приводить лише тильною стороною при деякій абдукції плеча (за рахунок надостной м'язи) - рука ' сурмача '. Активна функція якої м'язи відсутній? A 7-year-old child CANNOT retract his shoulder and raise it to the horizontal level, brings his hand to the face only with the back side with some abduction of the shoulder (due to the supraspinatus muscle) - 'trumpeter's' hand. The active function of which muscle is missing?

подостной podostnoy

Малої круглої Small Round

дельтоподібними deltoid

Великий круглої Big Round

Великий грудної Big chest

50 / 200
З нітратів, нітритів і нітрозамінів в організмі утворюється азотистая кислота, яка обумовлює окисне дезамінування азотистих про снованіі нуклеотидів. Це може привести до точкової мутації - заміни цитозину на: Nitrous acid is formed in the body from nitrates, nitrites and nitrosamines, which causes the oxidative deamination of nitrogenous nucleotides. This can lead to a point mutation - the replacement of cytosine with:

Аденін Adenine

Инозин Inosine

Тимин Timin

Урацил Uracil

гуанін guanine

51 / 200
Мати виявила у 5 річної дочки на періанальних складках білих ' черв'ячків ',' які викликали у неї свербіння і занепокоєння, і доставила їх в лабораторію. При огляді лікар побачив білих гельмінтів 0,5-1 см завдовжки, ниткоподібної форми з загостреними кінцями, у деяких вони закручені. Який діагноз можна поставити? The mother discovered white 'worms' on the perianal folds of her 5-year-old daughter, which caused her itching and anxiety, and took them to the laboratory. During the examination the doctor saw white helminths 0.5-1 cm long, filamentous in shape with pointed ends, some of them are twisted. What diagnosis can be made?

Ентеробіоз Enterobiosis

Аскаридоз Ascariasis

Описторхоз Opistorchosis

Теніоз Taeniosis

Дифиллоботриоз Diphyllobothriasis

52 / 200
У людини діагностовано галактоземія - хвороба накопичення. Внаслідок порушення якої клітинної структури виникла ця хвороба? A person has been diagnosed with galactosemia - an accumulation disease. As a result of a violation of which cellular structure, this disease arose?

Лізосом Lysosome

Клітинного центру Cell Center

Центросоми Centrosomes

Мітохондрій Mitochondrion

Комплексу Гольджі Golgi complex

53 / 200
При дисбактеріозах, що супроводжуються процесом гниття (протей, псевдомонади) і підвищенням рН фекалій, необхідно призначати біологічні препарати, Подкисляющие середу і проявляють антагоністичну дію. Які мікроорганізми для цього підходять? In case of dysbacteriosis accompanied by the process of decay (proteus, pseudomonads) and an increase in the pH of feces, it is necessary to prescribe biological preparations that acidify the environment and have an antagonistic effect. What microorganisms are needed for this suitable?

Клебсієли Klebsiels

ентеробактерій enterobacterium

серрацій serrations

азотобактер azotobacter

Біфідумбактеріі Bifidumbacterii

54 / 200
Після введення жабі стрихніну вона на мінімальне роздратування відповідає генералізованими судомами. Блокада який структури ЦНС є причиною цього? After strychnine is administered to a frog, it responds to minimal irritation with generalized convulsions. Blockade of which structure of the central nervous system is the cause of this?

Клітин Реншоу Klytin Renshaw

Гальмівних синапсів Inhibitory synapses

холінорецептори cholinergic receptors

збуджує синапсів excites synapses

адренорецептори adrenoceptors

55 / 200
Хворому з підозрою на черевний тиф лікар - інфекціоніст призначив бактеріологічне дослідження крові. Доцільність цього призначення пояснюється тим, що в першу тиждень захворювання тифо - паратифами спостерігається: To a patient suspected of having typhoid fever, an infectious disease doctor ordered a bacteriological blood test. The feasibility of this appointment is explained by the fact that in the first week of the disease, typhoid - paratyphoid fever is observed:

Вірусемія Virusemia

токсінеміі toxinemia

Бактериемия Bacteremia

Септицемія Septicemia

Септикопіємія Septicopyemia

56 / 200
У жінки 32 років з безсимптомним перебігом хвороби у другій раз народився мертвий дитина з вираженою мікроцефалією. Про якому захворюванні в першу чергу слід подумати лікаря? A 32-year-old woman with an asymptomatic course of the disease gave birth to a dead child with severe microcephaly for the second time. What disease should the doctor think about first?

Лістеріоз Listeriosis

Бруцельоз Brucellosis

Гістоплазмоз Histoplasmosis

Сифіліс Syphilis

Токсоплазмоз Toxoplasmosis

57 / 200
Для попередження нападу бронхіальної астми лікар призначив хворому кромолин натрію. Який з наведених механізмів характерний для цього кошти? To prevent an attack of bronchial asthma, the doctor prescribed cromolyn sodium to the patient. Which of the following mechanisms is characteristic of this remedy?

Зв'язування вільного гістаміну Binding of free histamine

Стабілізація мембран огрядних кліток Stabilization of fat cell membranes

Блокада гістамінових рецепторів Blockade of histamine receptors

Зниження концентрації імуноглобулінів Decrease in the concentration of immunoglobulins

Інактивація гістаміну Histamine inactivation

58 / 200
Дівчинка 10 років часто хворіє на гострі респіраторні інфекції, після яких спостерігаються множинні точкові крововиливи я в місцях тертя одягу. Гіповітаміноз якого вітаміну має місце у дівчинки? A 10-year-old girl often suffers from acute respiratory infections, after which multiple point hemorrhages are observed in places of friction of clothes. Hypovitaminosis of which vitamin occurs in the girl?

В 6 In 6

A A

B2 B2

C C

B1 B1

59 / 200
Людина отримав електротравму. При цьому струм пройшов через серцевий м'яз. Які небезпечні порушення в роботі серця, що вимагають невідкладного втручання, можуть виникнути в цій ситуації? A person received an electric shock. At the same time, the current passed through the heart muscle. What dangerous disturbances in the work of the heart that require immediate intervention can occur in this situation?

Екстрасистолія Extrasystole

Фібриляція передсердь Atrial fibrillation

Тахікардія Tachycardia

Брадикардія Bradycardia

Фибрилляция шлуночків Ventricular fibrillation

60 / 200
З допомогою шпателя зроблений зішкріб слизової рота людини. У незруйнованих епітеліальних клітинах пофарбованого мазка добре видно овальні ядра, неоднакові по розмірах. Яким шляхом відбувалося поділ цих клітин? Using a spatula, a scraping of the mucous membrane of a person's mouth was made. In the intact epithelial cells of the stained smear, oval nuclei of different sizes are clearly visible. How did the division of these cells occur?

Мейоз Meiosis

Амитоз Amitosis

Митоз Mitosis

Шизогонія Schizogony

Бінарне розподіл Binary Distribution

61 / 200
Під час ЕКГ - дослідження хворий виявлено періодичне поява шлуночкової екстрасистоли. При цьому встановлено, що перед екстрасистолою відсутня зубець Е. Яка найбільш ймовірна причина його зникнення? - Виникнення рефрактерного періоду в шлуночках During the ECG - examination of the patient, the periodic appearance of ventricular extrasystole was detected. At the same time, it was established that the E wave is absent before the extrasystole. What is the most likely reason for its disappearance? - The occurrence of refractory period in the ventricles

Блокада проведення імпульсу по передсердях Atrial conduction block

Виникнення рефрактерного періоду в передсерді Emergence of a refractory period in the atrium

Неможливість ретроградного проведення через AV-вузол Impossibility of retrograde conduction through the AV node

Блокада імпульсу в синусовому у злі Impulse blockade in sinus in evil

62 / 200
54- річний чоловік скаржиться на відсутність чутливості шкіри нижнього століття, зовнішньої латеральної поверхні носа, верхньої губи. Лікар при огляді встановлює запалення другої гілки трійчастого нерва. Через якийсь отвір виходить з черепа ця гілка? A 54-year-old man complains about the lack of sensitivity of the skin of the lower eyelid, the outer lateral surface of the nose, and the upper lip. During the examination, the doctor establishes inflammation of the second branch of the trigeminal nerve. Through some hole does this branch come out of the skull?

Овальний отвір Oval hole

Кругле отвір Round hole

Остистий отвір Prickly Hole

рвані отвір torn hole

Верхня глазничная щілину Upper orbital fissure

63 / 200
Хворий з діагнозом вогнищевого туберкульозу верхньої частки правого легкого отримує ізоніазид в складі комбінованої терапії. Через деякий час пацієнт почав скаржитися на м'язову слабкість, зниження чутливості шкіри, порушення зору, координації рухів. Який вітамінний пре парат доцільно використовувати для усунення цих явищ? A patient with a diagnosis of focal tuberculosis of the upper lobe of the right lung receives isoniazid as part of combined therapy. After some time, the patient began to complain of muscle weakness, decreased skin sensitivity, sight, coordination of movements. What vitamin preparation is appropriate to use to eliminate these phenomena?

Вітамін В6 Vitamin B6

Вітамін D Vitamin D

Вітамін С Vitamin C

Вітамін В12 Vitamin B12

Вітамін А Vitamin A

64 / 200
Хворий скаржиться на диспептичні розлади, мелену, гемороїдальні кровотечі. При дослідженні виявлено розширення сітки венозних судин на передній стінці живота в комплексі з збільшенням його розмірів. Яка патологія шлунково-кишкового тракту може проявитися такими симптомами? The patient complains of dyspeptic disorders, melena, hemorrhoidal bleeding. The examination revealed an expansion of the network of venous vessels on the front wall of the abdomen in combination with an increase in its size. What is the pathology of the gastrointestinal the intestinal tract can manifest itself with such symptoms?

Кишкова аутоинтоксикация Intestinal autointoxication

Виразкова хвороба Ulcer disease

Коліт Colitis

Портальная гіпертензія Portal hypertension

Ентерит Enteritis

65 / 200
У хворого з гіпертонічною хворобою при систематичному лікуванні антигіпертензивним засобом з'явився кашель. Яке з названих засобів може бути причиною даного побічного ефекту? A patient with hypertension developed a cough during systematic treatment with an antihypertensive agent. Which of the named agents can be the cause of this side effect?

Діхлотіазід Dichlorothiazide

Еналаприл Enalapril

Празозин Prazozin

Верапаміл Verapamil

Клофелін Clofelin

66 / 200
У експерименті досліджували поріг сили подразнення клітин різних тканин. В яких з наступних клітин він виявиться найнижчими? In the experiment, the threshold of the force of irritation of cells of various tissues was studied. In which of the following cells will it be the lowest?

В кардіоміоцитах In cardiomyocytes

В мотонейронах спинного мозку In motoneurons of the spinal cord

У міоцитах скелетних м'язів In skeletal muscle myocytes

У залізистих клітинах In iron cells

У міоцитах гладкких м'язів In smooth muscle myocytes

67 / 200
У хворого, які тривалий час брав глюкокортикоїди, в результаті відміни препарату виникло загострення наявного захворювання, зниження артеріального тиску, слабкість. Чим можна пояснити ці явища? A patient who had been taking glucocorticoids for a long time, as a result of withdrawal of the drug, there was an exacerbation of the existing disease, a decrease in blood pressure, weakness. How can these phenomena be explained?

звикання до препарату drug addiction

сенсибілізація sensitization

Виникненням недостатності надниркових залоз The occurrence of adrenal insufficiency

кумуляції cumulations

гіперпродукція АКТГ hyperproduction of ACTH

68 / 200
У психіатричну клініку доставлений хворий 40 років в стані збудження, агресії, марення. Який препарат слід ввести хворому? A 40-year-old patient was brought to a psychiatric clinic in a state of excitement, aggression, delirium. What drug should be administered to the patient?

Настоянку валеріани Valerian Tincture

Седуксен (діазепам) Seduxen (diazepam)

Натрію бромід Sodium bromide

Резерпін Reserpin

Аміназин Aminazine

69 / 200
Хворий госпіталізований з скаргами на блювоту, запаморочення, двоїння в очах, утруднене ковтання. Лікар запідозрив ботулізм. Які методи діагностики доцільно використовувати для підтвердження діагнозу? The patient is hospitalized with complaints of vomiting, dizziness, double vision, difficulty swallowing. The doctor suspected botulism. What diagnostic methods should be used to confirm the diagnosis?

Біологічну пробу, бактеріологічний Biological test, bacteriological

Протозоологіческій, мікроскопічний Protozoological, microscopic

- -

Бактеріологічний, мікологічний Bacteriological, mycological

Алергійну пробу, серологічний Allergy test, serological

70 / 200
У хворого нефритом виявлена глюкозурія і аминоацидурия. Порушення якого механізму реабсорбції глюкози і амінокислот є причиною цього? Glucosuria and aminoaciduria were detected in a patient with nephritis. Which mechanism of reabsorption of glucose and amino acids is the cause of this?

Простий дифузії Simple Diffusion

фагоцитозу phagocytosis

Вторинного Na+ - залежного транспорту Secondary Na+ - dependent transport

Первинного активного транспорту Primary active transport

піноцитоз pinocytosis

71 / 200
У хворого 27 років виявлені патологічні зміни печінки і головного мозку. У плазмі крові виявлено різке зниження, а в сечі збільшення вмісту міді. Поставлено діагноз - хвороба Вільсона. Активність якого ферменту в сироватці крові необхідно дослідити для підтвердження діагнозу? A 27-year-old patient was found to have pathological changes in the liver and brain. A sharp decrease in the blood plasma and an increase in the copper content in the urine was detected. The diagnosis was Wilson's disease. Activity which enzyme in the blood serum should be investigated to confirm the diagnosis?

Алкогольдегідрогенази Alcohol dehydrogenase

Карбоангідрази Carbonic anhydrases

лейцінамінопептідази leucine aminopeptidases

ксантиноксидазу xanthine oxidase

Церулоплазміну Ceruloplasmin

72 / 200
У юнака під час фізичної навантаження хвилинне споживання кисню і хвилинне виділення вуглекислого газу дорівнюють 1000 мл. Які субстрати окислюються в клітинах його організму? In a young man, during physical exertion, the minute consumption of oxygen and the minute release of carbon dioxide are equal to 1000 ml. What substrates are oxidized in the cells of his body?

Вуглеводи і білки Carbohydrates and proteins

Вуглеводи і жири Carbohydrates and fats

Вуглеводи Carbohydrates

Білки Proteins

Жири Fats

73 / 200
У гастроентерологічне відділення потрапив хворий 57 років з підозрою на синдром Золінгера - Еллісона, про що свідчило різке збільшення рівня гастрину в сироватці крові. Яке порушення секреторної функції шлунка найбільш імовірно? A 57-year-old patient was admitted to the gastroenterology department with suspicion of Zollinger-Ellison syndrome, which was evidenced by a sharp increase in the level of gastrin in the blood serum. What is the most likely violation of the secretory function of the stomach ?

Ахілія Achillia

Гиперсекреция гіпоацидного Hypersecretion of hypoacid

Гиперсекреция гіперацидних Hypersecretion of hyperacidic

Гіпосекреція гіперацидних Hyposecretion of hyperacidic

Гіпосекреція гіпоацидного Hyposecretion of hypoacid

74 / 200
Дитина 5 років поступив в ЛОР - відділення з діагнозом 'гнійне запалення середнього вуха.' Захворювання почалося з запалення носоглотки. Через якийсь канал скроневої кістки інфекція потрапила в барабанну порожнину? A 5-year-old child was admitted to the ENT department with a diagnosis of 'purulent inflammation of the middle ear.' The disease began with inflammation of the nasopharynx. Through some channel of the temporal bone, the infection entered the tympanic cavity?

Сонний канал Sleepy Channel

Мишечнотрубний канал Muscular tube channel

Барабанний каналець Tympanic tube

канадець барабанної струни drum string Canadian

Сонно - барабанні канальці Drowsy - tympanic tubules

75 / 200
При видаленні зуба для знеболювання використовують розчин новокаїну. З якої метою його вводять НЕ в ясна біля зуба, а в область проходження чутливого нерва? When removing a tooth, a solution of novocaine is used for pain relief. For what purpose is it injected NOT into the gums near the tooth, but into the area where the sensitive nerve passes?

Підвищується збудливість больових рецепторів The excitability of pain receptors increases

Порушується формування потенціалу дії больових рецепторів The formation of the action potential of pain receptors is disturbed

Пригноблюється аксональний транспорт Axonal transport is inhibited

Змінюється рН тканин в області анестезії The pH of tissues in the area of anesthesia changes

Блокується проведення больових імпульсів The conduction of pain impulses is blocked

76 / 200
У хворого сухим плевритом вислуховується шум тертя плеври. При ураженні якого виду епітелію відзначається цей симптом? A pleural friction noise is heard in a patient with dry pleurisy. What kind of epithelium is affected by this symptom?

Перехідний епітелій Transitional epithelium

багатошарове ий епітелій multilayered epithelium

Одношаровий кубічний епітелій Single-layer cuboidal epithelium

Одношаровий плоский епітелій Single-layer squamous epithelium

Одношаровий призматичний епітелій Single-layer prismatic epithelium

77 / 200
метильних групи (-СН3) використовуються в організмі для синтезу таких важливих сполук як креатин, холін, адреналін і ін. Яка з незамінних амінокислот, є джерелом цих груп є одна? methyl groups (-CH3) are used in the body for the synthesis of such important compounds as creatine, choline, adrenaline, etc. Which of the essential amino acids is the source of these groups is alone?

Валін Valin

Ізолейцин Isoleucine

Триптофан Tryptophan

Метіонін Methionine

Лейцин Leucine

78 / 200
При розтині померлого дитини, який страждав діареєю, виявлено ексікоз і розлитої фібринозний коліт. В мазку - відбитку слизової оболонки виявлені грамнегативні палички. Який найбільш ймовірний діагноз? During the autopsy of a dead child who suffered from diarrhea, exicosis and diffuse fibrinous colitis were found. Gram-negative bacilli were found in the smear - an impression of the mucous membrane. What is the most likely diagnosis?

Холера Cholera

Черевний тиф Typhoid

Сальмонельоз Salmonellosis

Дизентерія Dysentery

Стафилококковая кишкова інфекція Staphylococcal intestinal infection

79 / 200
Поступив з їжею глікоген Гідролізований в шлунково - кишковому тракті. Який кінцевий продукт утворився в результаті цього процесу? Glycogen entered with food Hydrolyzed in the gastrointestinal tract. What end product was formed as a result of this process?

Лактоза Lactose

Лактат Lactate

Фруктоза Fructose

Глюкоза Glucose

Галактоза Galactose

80 / 200
При обстеженні хворого виявлено новоутворення в білому речовині півкуль великого мозку з локалізацією в коліні і передньому відділі задньої ніжки внутрішньої капсули. Волокна якого провідного шляху мозку будуть зруйновані? During the examination of the patient, a neoplasm was found in the white matter of the cerebrum hemispheres with localization in the knee and the front part of the posterior leg of the internal capsule. The fibers of which conducting path of the brain will be destroyed?

Tr. frontopontinus Tr. frontopontinus

Tr. pyramidalis Tr. pyramidalis

Tr. parietooccipitopontinus Tr. parietooccipitopontinus

Tr. frontothalamicus Tr. frontothalamicus

Tr. thalamocorticalis Tr. thalamocorticalis

81 / 200
У людини внаслідок довільної затримки дихання на 40 с зросли частота серцевих скорочень і системне артеріальний тиск. Реалізація будь механізмів регуляції зумовила зміна показників? As a result of arbitrary breath holding for 40 s, heart rate and systemic blood pressure increased in a person. Was the change in indicators caused by the implementation of any regulation mechanisms?

Умовні парасимпатичні рефлекси - Conditional parasympathetic reflexes -

Безумовні парасимпатичні рефлекси Unconditioned parasympathetic reflexes

Умовні симпатичні рефлекси Conditional sympathetic reflexes

Безумовні симпатичні рефлекси Unconditioned sympathetic reflexes

82 / 200
На протязі двох тижнів хвора приймала мікстуру, прописану невропатологом по приводу неврастенії. Самопочуття хворої декілька покращилось, однак незабаром з'явилися скарги на нежить, кон'юнктивіт, шкірну висип, млявість і ослаблення пам'яті. Був встановлений діагноз ' бромізм '. Що доцільно призначити для послаблення симптомів? For two weeks, the patient took a mixture prescribed by a neurologist for neurasthenia. The patient's well-being improved somewhat, but soon complaints of runny nose, conjunctivitis, skin rash, lethargy and memory loss. A diagnosis of bromism was made. What should be prescribed to relieve the symptoms?

- -

Полиглюкин Polyglukin

Розчин глюкози 5% Glucose solution 5%

Натрію хлорид Sodium chloride

Аспаркам Asparkam

83 / 200
Хворий НЕ може підняти опущену вниз нижню щелепу. Які м'язи НЕ можуть виконати свої функції? The patient CANNOT raise the lowered lower jaw. Which muscles CANNOT perform their functions?

Кругова м'яз рота Round of mouth

надчерепной м'яз supracranial muscle

Жувальні м'язи Masticatory muscles

М'яз, що піднімає кут рота Muscle that raises the corner of the mouth

Мімічні м'язи Mimic muscles

84 / 200
До лікаря звернувся хворий з пошкодженої стопою, забинтованою брудною марлевою пов'язкою, просоченої гнійними виділеннями. Спроба зняти пов'язку для огляду і обробки рани викликає гостру біль, так як вона прилипла до рани поверхні. Який антисептик полегшить видалення пов'язки і очистить рану від бруду і гною? A patient came to the doctor with an injured foot, bandaged with a dirty gauze bandage soaked with purulent secretions. An attempt to remove the bandage to examine and treat the wound causes sharp pain, as it stuck to the surface of the wound. What antiseptic will facilitate the removal of the bandage and clean the wound of dirt and pus?

Перекис водню Hydrogen peroxide

Фурацилин Furacilin

Етакрідін Ethacridine

Етоній Ethonia

Калію перманганат Potassium permanganate

85 / 200
Хворому з непереносимістю антибіотиків для лікування пневмонії призначений сульфален. Через кілька днів у хворого розвинувся гемоліз еритроцитів. Недостатність якого ферменту в організмі хворого сприяла розвитку цього побічного ефекту? Sulfalene was prescribed to a patient with antibiotic intolerance to treat pneumonia. After a few days, the patient developed hemolysis of erythrocytes. The deficiency of which enzyme in the patient's body contributed to the development of this side effect?

N-ацетилтрансферази N-acetyltransferases

ацетальдегіддегідрогенази acetaldehyde dehydrogenases

Урідіндіфосфатглюкуроновой трансферази Glucuronyl diphosphate transferase

холінестерази cholinesterases

Глюкозо-6-фосфатдегідрогенази Glucose-6-phosphate dehydrogenase

86 / 200
До лікаря звернувся хворий з скаргами на постійну спрагу. Виявлено гіперглікемія, поліурія та підвищений змісту 17- кетостероїдів в сечі. Яке захворювання найбільш ймовірно? A patient came to the doctor with complaints of constant thirst. Hyperglycemia, polyuria, and increased content of 17-ketosteroids in the urine were detected. What disease is most likely?

Глікогеноз I типу Glycogenosis type I

Стероїдний діабет Steroid diabetes

Інсулінозалежний діабет Insulin-dependent diabetes

Аддисонова хвороба Addison's disease

Мікседема Myxedema

87 / 200
При рентгенологічному дослідженні кісток основи черепа виявлено збільшення порожнини турецького сідла, витончення передніх похилих відростків, руйнування різних ділянок турецького сідла. Пухлина який ендокринної залози може викликати таке руйнування кісток? An X-ray examination of the bones of the base of the skull revealed an increase in the cavity of the sella turcica, thinning of the anterior oblique processes, destruction of various areas of the sella turcica. A tumor of which endocrine gland can cause such destruction of bones?

вилочкова залози thymus

Наднирників Adrenals

щитовидної залози thyroid

Епіфіза Pinephysis

Гіпофіза Pituitary

88 / 200
Людина стоїть в кімнаті в легкій одязі, температура повітря +14°С. Вікна та двері закриті. Яким шляхом він віддає більше всього тепла? A person is standing in a room in light clothes, the air temperature is +14°С. The windows and doors are closed. Which way does he give off the most heat?

перспірація perspiration

Теплорадіація Thermal radiation

Теплопроведение Heat conduction

Випаровування Evaporation

Конвекція Convection

89 / 200
Клінічні дослідження крові рекомендується проводити натще і вранці. Які зміни компонентів крові можливі, якщо провести забір крові після прийому їжі? Clinical blood tests are recommended to be performed on an empty stomach and in the morning. What changes in blood components are possible if blood is taken after eating?

Зниження числа еритроцитів Decreasing the number of erythrocytes

Зниження числа тромбоцитів Decreased platelet count

Збільшення білків плазми Increase in plasma proteins

Збільшення числа лейкоцитів Increase in the number of leukocytes

Збільшення числа еритроцитів Increase in the number of erythrocytes

90 / 200
Після фізичної навантаження підвищився артеріальний тиск крові. Чому? After exercise, blood pressure increased. Why?

Збільшилася кількість гемоглобіну Hemoglobin increased

Збільшилася кількість функціонуючих капілярів The number of functioning capillaries has increased

Збільшилася зміст іонів калію в плазмі крові The content of potassium ions in blood plasma increased

Збільшився хвилин ний обсяг кровообігу The minute volume of blood circulation has increased

Збільшилася зміст води в плазмі крові The content of water in blood plasma has increased

91 / 200
У хворого з запаленням легенів непереносимість антибіотиків. Який з комбінованих сульфаніламідних препаратів слід призначити хворому? A patient with pneumonia has intolerance to antibiotics. Which of the combined sulfonamide drugs should be prescribed to the patient?

Стрептоцид Streptocide

Етазол Ethazol

Сульфадиметоксин Sulfadimethoxine

Бісептол Biseptol

Сульфацил натрію Sulfacyl sodium

92 / 200
Тривале перебування в умовах спеки викликало у людини спрагу. Сигналізація від яких рецепторів, в першу чергу, зумовила її розвиток? Prolonged stay in hot conditions made a person thirsty. Signaling from which receptors primarily determined its development?

барорецепторами дуги аорти baroreceptors of the aortic arch

Осморецептори гіпоталамуса Osmoreceptors of the hypothalamus

натрієві рецепторів гіпоталамуса hypothalamus sodium receptors

глюкорецептори гіпоталамуса glucoreceptors of the hypothalamus

осморецептори печінки liver osmoreceptors

93 / 200
У юнака 20 років травмовано праве яєчко. Яку небезпеку це становить для лівого (здорового) яєчка? A 20-year-old boy has an injury to his right testicle. What danger does it pose to the left (healthy) testicle?

Розвиток инфекц іонного процесу Development of the infectious process

Розвиток гіпертрофії Development of hypertrophy

Розвиток атрофії Development of atrophy

Демаскування антигену і виникнення пошкодження антитілами Antigen unmasking and antibody damage

Чи не загрожує нічим Doesn't threaten anything

94 / 200
При обстеженні хворого визначається наявність гіперглікемії, кетонурія, поліурії, гиперстенурия і глюкозурії. Яка форма порушення кислотно - лужного балансу має місце в даній ситуації? When examining the patient, the presence of hyperglycemia, ketonuria, polyuria, hypersthenuria, and glucosuria is determined. What form of acid-alkaline imbalance occurs in this situation?

Негазовий алкалоз Non-gas alkalosis

Газовий алкалоз Gas alkalosis

Газовий ацидоз Gas acidosis

Метаболічний ацидоз Metabolic acidosis

Метаболічний алкалоз Metabolic alkalosis

95 / 200
У хворого 45 років на правій нозі спостерігається блідість шкіри гомілки і стопи, визначається відсутність пульсації тильної артерії стопи та задньої великогомілкової артерії. Пульсація стегнової артерії збережена. Поранення який артерії сталося? A 45-year-old patient has pallor of the skin of the lower leg and foot on the right leg, the absence of pulsation of the posterior artery of the foot and the posterior tibial artery is determined. The pulsation of the femoral artery is preserved. Injury of which artery happened?

Зовнішньої клубової External iliac

Глибокої артерії стегна Deep femoral artery

Підколінної Popular

Низхідній колінної - Малогомілкової Lower patella - Calf

96 / 200
Чоловікові 70 років, яка страждає хронічним бронхітом, призначений противокашлевой препарат - кодеїн. Який механізм забезпечує противокашлевой ефект? A 70-year-old man suffering from chronic bronchitis is prescribed an antitussive drug - codeine. What mechanism provides the antitussive effect?

Рефлекторний Reflex

Конкурентний Competitive

Місцеве дію Local action

Центральний Central

Периферичний дію Peripheral activity

97 / 200
Людина сидів у кріслі з закритими очима, коли задзвонив телефон. Як зміниться у нього ЕЕГ? A person was sitting in a chair with his eyes closed when the phone rang. How will his EEG change?

Альфа - ритм зміниться на бета - ритм Alpha rhythm will change to beta rhythm

Збільшиться тета - ритм Theta rhythm will increase

Тета - ритм зміниться на альфа - ритм Theta rhythm will change to alpha rhythm

Збільшиться альфа - ритм The alpha rhythm will increase

Бета - ритм зміниться на тета - ритм Beta rhythm will change to theta rhythm

98 / 200
У приймальне відділення доставлений хворий у непритомному стані. Шкіра холодна, зіниці звужені, дихання з утрудненням, відзначається періодичність по типу Чейна - Стокса, артеріальний тиск знижений, сечовий міхур переповнений. Отруєння яким речовиною най більш ймовірно? A patient was brought to the reception department in an unconscious state. The skin is cold, the pupils are narrowed, breathing is difficult, periodicity according to the Cheyne-Stokes type is noted, blood pressure is low, the bladder overcrowded. Poisoning by what substance is most likely?

транквілізатори tranquilizers

ненаркотичних анальгетиками non-narcotic analgesics

- -

Наркотичними анальгетиками Narcotic analgesics

М-холиноблокаторами M-cholinoblockers

99 / 200
У хворого гострим мієлобластний лейкоз виявлено: збільшення печінки, селезінки, анемія, мієлобласти в периферичної крові. Який принциповий ознака дозволяє відрізнити гострий мієлобластний лейкоз від хронічного? A patient with acute myeloblastic leukemia was diagnosed with: enlargement of the liver, spleen, anemia, myeloblasts in the peripheral blood. What is the main feature that allows you to distinguish acute myeloblastic leukemia from chronic?

Анемія Anemia

Владні клітини в периферичної крові Power cells in peripheral blood

Панцитопенія Pancytopenia

Тромбоцитопенія Thrombocytopenia

Лейкемічний провал Leukemic failure

100 / 200
При дослідженні крові хворого виявлено значне збільшення активності МВ - форм КФК (креатинфосфокінази) та ЛДГ-1. Яка найбільш ймовірна патологія? When examining the patient's blood, a significant increase in the activity of CF - forms of CK (creatine phosphokinase) and LDH-1 was revealed. What is the most likely pathology?

Ревматизм Rheumatism

Панкреатит Pancreatitis

Гепатит Hepatitis

Інфаркт міокарда Myocardial infarction

Холецистит Cholecystitis

101 / 200
Сульфаніламідні препарати нагадують по структурі парааминобензойную кислоту. В ніж з остоит молекулярна основа їх фармакологічного ефекту? Sulfonamide drugs resemble paraaminobenzoic acid in structure. What is the molecular basis of their pharmacological effect?

В порушення синтезу вітаміну In violation of vitamin synthesis

В активації ліполізу In activation of lipolysis

В руйнуванні клітинної мембрани In the destruction of the cell membrane

У ингибировании гліколізу In inhibition of glycolysis

У зв'язуванні з ДНК In binding with DNA

102 / 200
На прийом до лікаря прийшов пацієнт дуже високого зростання, з довгими товстими пальцями рук, великою нижньою щелепою і відвислою нижньою губою. Підвищену секрецію якого гормону який залози можна підозрювати? A very tall patient, with long thick fingers, a large lower jaw and a drooping lower lip, came to the doctor for an appointment. Increased secretion of which hormone can be suspected by which gland?'

Гормонів щитовидної залози Thyroid hormones

гонадотропні гормону передньої частки гіпофіза gonadotropic hormones of the anterior lobe of the pituitary gland

антидіуретичного гормону задньої частки гіпофіза antidiuretic hormone of the posterior lobe of the pituitary gland

Гормонів надниркової залози з групи глюкокортикоїдів Adrenal hormones from the group of glucocorticoids

Соматотропного гормону передньої частки гіпофіза Somatotropic hormone of the anterior lobe of the pituitary gland

103 / 200
Хворий 37 років, що страждає на облітеруючий ендартеріїтом судин нижніх кінцівок, отримує фенилин в добовій дозі 60 мг / кг. У зв'язку з проявами судомного синдрому (в анамнезі ЧМТ) призначений фенобарбітал, після відміни якого у хворого виникло носове кровотеча. Дане ускладнення пов'язане з: A 37-year-old patient suffering from obliterating endarteritis of the vessels of the lower extremities receives phenylin in a daily dose of 60 mg/kg. Due to the manifestations of the convulsive syndrome (in history of TBI) phenobarbital was prescribed, after the withdrawal of which the patient developed a nosebleed. This complication is associated with:

Індукцією феноба рбіталом ферментів мікросомального окислення в печінці Induction of phenob orbital by enzymes of microsomal oxidation in the liver

окислювальний дезаминированием фенилина oxidative deamination of phenylene

кон'югації фенилина з глюкуроновою кислотою conjugations of phenylene with glucuronic acid

аліфатичні гідроксилюванням фенобарбіталу aliphatic by hydroxylation of phenobarbital

Гальмуванням фенобарбіталом мікросомального окислення в печінці Inhibition of microsomal oxidation in the liver by phenobarbital

104 / 200
У хворого 30 років діагностовано ішемічна хвороба серця. Напади стенокардії турбують вже 3 роки. Виявлено спадковий характер захворювання. Який вид гиперлипопротеидемии найбільш ймовірно буде виявлено у хворого? A 30-year-old patient has been diagnosed with coronary heart disease. Angina attacks have been bothering him for 3 years. The hereditary nature of the disease has been revealed. What type of hyperlipoproteinemia is most likely to be detected in the patient?

IV (гіперпребеталіпопротеідемія). IV (hyperprebetalipoproteinemia).

I (гіперхіломікронемія). I (hyperchylomicronemia).

V (гіперпребеталіпопротеідемія і гіперхіломікронемія). V (hyperprebetalipoproteinemia and hyperchylomicronemia).

III (дісбеталіпопротеідемія). III (dysbetalipoproteinemia).

II (гіпербеталіпопротеідемія). II (hyperbetalipoproteinemia).

105 / 200
Жінка 53 років, зріст 163 см, вага 92 кг, рівномірне відкладення жиру, особа одутле, малорухлива, апатична. При натисканні шкіри ноги залишається ямка. Порушенням функції якої залози обумовлено стан хворої? Woman 53 years old, height 163 cm, weight 92 kg, uniform deposition of fat, face puffy, sedentary, apathetic. When the skin of the leg is pressed, a dimple remains. Violation of the function of which glands caused by the condition of the patient?

Паращитовидних Parathyroids

Гіпофіза Pituitary

Полових Sex

Щитоподібної Thyroid

надниркові adrenal

106 / 200
В приймальне відділення доставлений дитина 1,5 років з ознаками отруєння нітратами: стійкий ціаноз, задишка, судоми. Який пат огенетіческій механізм лежить в основі цих симптомів? A 1.5-year-old child was brought to the reception department with signs of nitrate poisoning: persistent cyanosis, shortness of breath, convulsions. What pathogenetic mechanism underlies these symptoms?

Освіта карбоксигемоглобина Education of carboxyhemoglobin

Освіта карбогемоглобін Carbohemoglobin Education

Освіта оксигемоглобина Education of oxyhemoglobin

Освіта скороченої гемоглобіну Education of reduced hemoglobin

Освіта метгемоглобіну Methemoglobin Education

107 / 200
Під час обіду дитина поперхнувся і аспірованої їжу. Почався сильний кашель, шкіра і слизові ціанотичні, пульс прискорений, дихання рідкісне, видих подовжений. Яке порушення зовнішнього дихання розвинулося у дитини? During lunch, the child sneezed and aspirated food. A strong cough began, the skin and mucous membranes were cyanotic, the pulse accelerated, breathing was rare, exhalation was prolonged. What disorder of external breathing developed in child?

Стадія инспираторной задишки при асфіксії Stage of inspiratory dyspnea in asphyxia

Дихання Біота Biot's Breath

Стадія експіраторной задишки при асфіксії Stage of expiratory dyspnea in asphyxia

Стенотичне дихання Stenotic breathing

альтернирующая дихання alternating breathing

108 / 200
У померлого від малярії виражена жовтушність шкіри, склер та слизових оболонок. При розтині: селезінка збільшена в розмірі, аспидно - сірого кольору. Аспидно - сіра забарвлення селезінки обуслов лена наявністю: The person who died of malaria had pronounced yellowness of the skin, sclera, and mucous membranes. At autopsy: the spleen was enlarged in size, aspid-gray in color. The aspid-gray color of the spleen was caused by presence:

Гемопорфіріна Hemoporphyrin

Меланіну Melanin

гемомеланін hemomelanin

гемосидерину hemosiderin

ліпофусцин lipofuscin

109 / 200
Хворому з маніакально - депресивним синдромом в стадії депресії, скаржилися на відчуття тривоги, страху, був призначений антидепресант з супутнім псіхоседатівним ефектом. Який це був препарат? A patient with manic-depressive syndrome in the stage of depression, complained of a feeling of anxiety, fear, was prescribed an antidepressant with a concomitant psychosedative effect. What was this drug?

Ніаламід Nialamide

Имизин (іміпрамін) Imisin (imipramine)

Сіднофен Sidnofen

інказан inkazan

Амитриптилин Amitriptyline

110 / 200
Який препарат слід призначити хворому, у якого з - за висипу алергічного характеру з почервонінням, набряком, сильним свербінням з'явилася безсоння? What drug should be prescribed to a patient who has insomnia due to an allergic rash with redness, swelling, and severe itching?

Димедрол Diphenhydramine

Фенобарбітал Phenobarbital

Нітразепам Nitrazepam

Хлоралгідрат Chloral hydrate

Натрію оксибутират Sodium oxybutyrate

111 / 200
З метою перевірки крові донорів на наявність антигенів гепатиту B необхідно застосовувати високочутливі методи. Яку з названих реакцій слід застосує ь з вказаною метою? In order to test the blood of donors for the presence of hepatitis B antigens, it is necessary to use highly sensitive methods. Which of the named reactions should be used for this purpose?

Реакцію непрямої імунофлуоресценції Indirect immunofluorescence reaction

Реакцію непрямої гемаглютинації Indirect hemagglutination reaction

Іммуноелектрофорез Immunoelectrophoresis

Твердофазний імуноферментний аналіз Solid-phase enzyme-linked immunosorbent assay

Реакцію зв'язування комплементу Complement binding reaction

112 / 200
У пацієнта після травми виникли паралічі, розлади больової чутливості справа ; зліва паралічі відсутні, але порушена больова і температурна чутливість. Яка причина такого явища? After the injury, the patient developed paralysis, disorders of pain sensitivity on the right; on the left, there is no paralysis, but pain and temperature sensitivity is disturbed. What is the cause of this phenomenon?

Одностороннє ураження спинного мозку з правої сторони Unilateral spinal cord injury on the right side

Пошкодження рухової зони кори головного мозку Damage of the motor cortex of the brain

Пошкодження мозочка Cerebellar damage

Пошкодження стовбура мозку Brain stem damage

Пошкодження середнього мозку Midbrain damage

113 / 200
При обстеженні хворого встановлено діагноз - кліщовий поворотний тиф. Яким шляхом міг заразитися хворий? During the examination of the patient, a diagnosis of tick-borne relapsing typhus was established. How could the patient become infected?

Через укус тайгового кліща Due to a taiga tick bite

Через укус москіта Because of a mosquito bite

Через укус малярійного комара Due to a malaria mosquito bite

Через укус селищного кліща Due to the bite of the village tick

Через укус собачого кліща Due to dog tick bite

114 / 200
В шкірі виявлена щільна, рухома, чітко відмежована від оточуючих тканин пухлина. У розрізі вона білого кольору, представлена волокнистою тканиною. Мікроскопічно - хаотично переплетені колагенові волокна, клітин мало. Яка це пухлина? A dense, mobile tumor, clearly separated from the surrounding tissues, was detected in the skin. In section, it is white in color, represented by fibrous tissue. Microscopically, collagen fibers are chaotically intertwined, there are few cells . What kind of tumor is this?

Десмоїд Desmoid

Дерматофіброма Dermatofibroma

гістіоцитома histiocytoma

Тверда фіброма Solid fibroma

М'яка фіброма Soft fibroma

115 / 200
В печінці хворого порушена детоксикація природних метаболітів і ксенобіотиків. Активність якого цитохрому може бути знижена? Detoxification of natural metabolites and xenobiotics is disturbed in the patient's liver. The activity of which cytochrome can be reduced?

Цитохроми Р-450 Cytochromes P-450

цитохроми З-1 cytochromes C-1

цитохромоксидази cytochrome oxidases

цитохроми cytochromes

Гемоглобіну Hemoglobin

116 / 200
У хворого тромбоз коронарних судин. Необхідно введення речовини, що відновлює їх прохідність. Яке з пропонованих речовин про ладает необхідними властивостями? The patient has thrombosis of coronary vessels. It is necessary to introduce a substance that restores their patency. Which of the proposed substances has the necessary properties?

Гепарин Heparin

Пентоксифілін Pentoxifylline

Стрептокиназа Streptokinase

Фенилин Feniline

Ацетилсаліцилова кислота Acetylsalicylic acid

117 / 200
При обстеженні хворого з гемофілією виявлено зміна деяких показників крові. Який з перерахованих ознак відповідає цьому захворюванню? When examining a patient with hemophilia, a change in some blood parameters was detected. Which of the listed signs corresponds to this disease?

афібриногенемією afibrinogenemia

Еритроцитоз Erythrocytosis

Час згортання крові загальмовано Blood clotting time is inhibited

Тромбоцитопенія Thrombocytopenia

Еозинофілія Eosinophilia

118 / 200
У хворого діагностовано гострий інфаркт міокарда, що супроводжується стійкими болями за грудиною. Неефективність раніше введених препаратів дала підставу лікарю провести нейролептанальгезию. Який нейролептик треба використовувати? The patient was diagnosed with an acute myocardial infarction, which is accompanied by persistent chest pains. The ineffectiveness of previously administered drugs gave the doctor reason to perform neuroleptanalgesia. Which neuroleptic should be used?

Аміназин Aminazine

Галоперидол Haloperidol

Дроперидол Droperidol

Резерпін Reserpin

Метаперазін Metaperazine

119 / 200
Відомо, що вірус імунодефіциту людини належить до сімейства ретровірусів. Вкажіть осно вной ознака, що характеризує дане сімейство. It is known that the human immunodeficiency virus belongs to the family of retroviruses. Specify the main feature that characterizes this family.

Відсутність інтеграції нуклеїнової кислоти в геном господаря Absence of nucleic acid integration into host genome

- -

Наявність мінус - РНК Presence minus - RNA

Виявлення антигенів реакцією імуноферментного аналізу Detection of antigens by enzyme immunoassay

Наявність ферменту зворотної транскриптази Presence of reverse transcriptase enzyme

120 / 200
При мікроскопічному дослідженні нефробіоптата виявлено наявність півмісяців більш ніж в 50% клубочків, капілярні петлі некротизованих, в просвіті їх виявлено фібринові тромби, виражений тубуло - інтерстиціальний компонент. Про якому захворюванні нирок слід думати? During microscopic examination of the nephrobioptate, crescents were found in more than 50% of the glomeruli, capillary loops were necrotic, fibrin thrombi were found in their lumen, and the tubulo-interstitial component was pronounced. About which kidney disease should be considered?

Ліпоїдний нефроз Lipoid nephrosis

Швидкопрогресуючий гломерулонефрит Rapidly progressive glomerulonephritis

Хронічний гломерулонефрит Chronic glomerulonephritis

Амилоидоз Amyloidosis

Некротический нефроз Necrotic nephrosis

121 / 200
Морфологічні дослідження селезінки виявили активізацію імунних реакцій в організмі. В яких структурах даного органу починається антигензависимая проліферація Т - лімфоцитів? Morphological studies of the spleen revealed the activation of immune reactions in the body. In which structures of this organ does the antigen-dependent proliferation of T lymphocytes begin?

Періартеріального зона білої пульпи Periarterial zone of the white pulp

Червона пульпа Red Pulp

Мантійна зона білої пульпи White Pulp Mantle Zone

Маргінальна зона білої пульпи Marginal zone of white pulp

Центральна зона білої пульпи Central zone of white pulp

122 / 200
При обстеженні хворого з травматичним пошкодженням головного мозку виявлено, що він перестав розрізняти переміщення предмета по шкірі. Який відділ кори мозку пошкоджений? When examining a patient with traumatic brain damage, it was found that he stopped distinguishing the movement of an object on the skin. Which part of the cerebral cortex is damaged?

Потилична частка кори Occipital cortex

Передня центральна звивина Anterior central gyrus

Тім'яна частка кори Parietal cortex

Лобова частка кори Frontal cortex

Задня центральна звивина Posterior central gyrus

123 / 200
Після внебольничного аборту у жінки прогресував гнійний ендоміометрит зі смертельним результатом. При розтині померлої виявлені численні абсцеси легких, субкапсулярні гнійники в нирках, гіперплазія селезінки. Яка форма сепсису виникла у хворої? After an out-of-hospital abortion, a woman developed purulent endomyometritis with a fatal outcome. At the autopsy of the deceased, numerous lung abscesses, subcapsular abscesses in the kidneys, and hyperplasia of the spleen were found. What form of sepsis arose in sick?

уросепсисі urosepsis

Легеневий сепсис Pulmonary sepsis

хроніосепсису chroniosepsis

Септицемія Septicemia

З ептікопіемія With epticopiemia

124 / 200
Хвора 45 років скаржиться на задишку при невеликій фізичної навантаженні, набряки на балках, в анамнезі часті ангіни, хворіє на протязі двох років. Діагностовано міокардит, комбінований мітральний порок серця, недостатність кровообігу. Який гемодинамічний механізм декомпенсації серця у хворої? A 45-year-old patient complains of shortness of breath with light physical exertion, swelling on the beams, a history of frequent angina, has been ill for two years. Myocarditis, combined mitral heart disease is diagnosed , circulatory insufficiency. What is the hemodynamic mechanism of heart decompensation in the patient?

Зменшення обсягу циркулюючої крові Decreased volume of circulating blood

Підвищення артеріального тиску Increased blood pressure

Зменшення хвилинного обсягу серця Decrease in cardiac output

Тахікардія Tachycardia

Зниження венозного тиску Reduction of venous pressure

125 / 200
У дитини 7 років на шкірі розгинальних поверхонь ліктьових і колінних суглобів з'явилися щільні, безболісні вузлики розміром 1-2 мм. У біоптаті вузликів - великий осередок фибриноидного некрозу сполучної тканини з лімфоцитами і макрофагами по периферії. При якому захворюванні спостерігаються такі вузлики? A 7-year-old child developed dense, painless nodules 1-2 mm in size on the skin of the extensor surfaces of the elbow and knee joints. In the biopsy of the nodules, there was a large focus of fibrinoid necrosis connective tissue with lymphocytes and macrophages on the periphery. In what disease are such nodules observed?

Системна червона вовчак Systemic lupus erythematosus

Склеродермия Scleroderma

Вузликовий періартеріїт Nodular periarteritis

Ревматоїдний артрит Rheumatoid arthritis

Ревматизм Rheumatism

126 / 200
Для лікування урогенітальних інфекцій використовують хінолони - інгібітори ферменту ДНК - гірази. Який процес порушується під впливом хінолонів в першу чергу? Quinolones are used to treat urogenital infections - inhibitors of the DNA gyrase enzyme. What process is disrupted by quinolones in the first place?

Зворотній транскрипція Reverse transcription

Ампліфікація генів Gene Amplification

Репарація Reparation

Реплікація Replication

Рекомбінація генів Gene recombination

127 / 200
При мікроскопічному дослідженні тканини печінки було виявлено, що деякі клітини розпалися на невеликі фрагменти з окремими органелами і залишками ядра, оточені мембраною. Запальна реакція відсутня. Для якого патологічного процесу характерні ці зміни? During microscopic examination of liver tissue, it was found that some cells disintegrated into small fragments with separate organelles and remnants of the nucleus surrounded by a membrane. There is no inflammatory reaction. For which pathological process are these changes typical?

Пла зморексіс Pla zmorexis

каріорексисом karyorrhexis

Апоптоз Apoptosis

Некроз Necrosis

Плазмоліз Plasmolysis

128 / 200
У хворого має місце позаклітинний набряк тканин (збільшені розміри м'яких тканин кінцівок, печінки і т. д.). Зменшення якого параметра гомеостазу є найбільш вірогідною причиною розвитку набряку? The patient has extracellular edema of tissues (increased sizes of soft tissues of the limbs, liver, etc.). A decrease in which parameter of homeostasis is the most likely cause of the development edema?

гематокриту hematocrit

Осмотичного тиску плазми крові Osmotic pressure of blood plasma

- -

в'язкості viscosities

Онкотичного тиску плазми крові Oncotic blood plasma pressure

129 / 200
При обстеженні юнака з розумовою відсталістю виявлено евнухоідний будова тіла, недорозвиненість статевих органів. У клітинах порожнини рота - статевий хроматин. Який метод генетичного дослідження слід використовувати для уточнення діагнозу? During the examination of a young man with mental retardation, a eunuchoid body structure and underdevelopment of the genitals were revealed. Sex chromatin is found in the cells of the oral cavity. What method of genetic research should be used to clarify the diagnosis?'

Біохімічний Biochemical

дерматогліфіки dermatoglyphics

Клініко-генеалогічний Clinical-genealogical

Цитологічний Cytological

Популяційно-статистичний Population-statistical

130 / 200
У людини зареєстрована електрокардіограма зі зниженою амплітудою зубця R. Що означає цей зубець на ЕКГ? A person has a registered electrocardiogram with a reduced amplitude of the R wave. What does this wave mean on the ECG?

Поширення збудження від передсердь до шлуночків Propagation of excitation from atria to ventricles

Електричну систолу серця Electric systole of the heart

Електричну діастолу серця Electric heart diastole

Поширення збудження по передсердям Propagation of excitation in the atria

Поширення збудження по шлуночках Propagation of excitation across the ventricles

131 / 200
У жінки 68 років після інсульту відсутні рухи в верхній і нижній правих кінцівках. Тонус м'язів цих кінцівок і рефлекси в них підвищені. Є патологічні рефлекси. Яка це форма паралічу? A 68-year-old woman after a stroke has no movements in her upper and lower right limbs. The muscle tone of these limbs and their reflexes are increased. There are pathological reflexes. What is this a form of paralysis?

моноплегии monoplegia

Параплегія Paraplegia

Тетраплегія Tetraplegia

Дисоціація Dissociation

Геміплегія Hemiplegia

132 / 200
У хворого 17 років інтраопераційно на нижній поверхні печінки виявлена пухлина розмірами 4,5x5,0x3,5 см з субсерозной локалізацією, темно - червоного кольору, на розрізі представлена порожнинами зі значним вмістом крові. До акой найбільш ймовірний попередній діагноз? In a 17-year-old patient, intraoperatively, a tumor measuring 4.5x5.0x3.5 cm with subserosal localization, dark red in color, represented by cavities on section was detected intraoperatively on the lower surface of the liver with significant blood content. What is the most likely preliminary diagnosis?

Гемангіоперицитома Hemangiopericytoma

Гемангіоендотеліома Hemangioendothelioma

Лимфангиома Lymphangioma

Капілярна гемангіома Capillary hemangioma

Кавернозная гемангіома Cavernous hemangioma

133 / 200
У хворого при гастроскопії виявлено недостатнє кількість слизу, що покриває слизову оболонку. З порушенням функції яких клітин стінки шлунка це пов'язано? The patient's gastroscopy revealed an insufficient amount of mucus covering the mucous membrane. This is due to a malfunction of which cells of the stomach wall?

B- клітин призматичного залозистого епітелію B cells of the prismatic glandular epithelium

ендокріноціти endocrinocytes

шеечную клітин neck of cells

Парієтальних клітин залоз шлунка Parietal cells of gastric glands

Головних екзокриноцитів Main exocrinocytes

134 / 200
Особам, які бажають схуднути, рекомендують включати в харчовий раціон побільше нежирної яловичини. Якими властивостями білків це пояснюється? People who want to lose weight are recommended to include more lean beef in their diet. What properties of proteins explain this?

Наявністю найбільшого специфічно-динамічного дії By the presence of the largest specific dynamic action

Швидким насиченням Fast saturation

Низькою калорійністю Low calorie

Тривалою затримкою в шлунку Prolonged delay in the stomach

Поганим всмоктуванням Poor suction

135 / 200
Чоловік чину 65 років, що страждає на подагру, скаржиться на болі в області нирок. При ультразвуковому дослідженні встановлено наявність ниркових каменів. В результаті якого процесу утворюються ниркові камені? A 65-year-old man suffering from gout complains of pain in the kidney area. An ultrasound examination revealed the presence of kidney stones. As a result of which process are kidney stones and stones formed ?

катаболізм білків protein catabolism

Розпаду гема Heme Decay

Розпаду пуринових нуклеотидів Decomposition of purine nucleotides

орнітінового циклу ornithine cycle

Відновлення цистеїну Cysteine Recovery

136 / 200
У хворого з діагнозом ' злоякісний карціноід ' різко підвищено вміст серотоніну в крові. З якої амінокислоти може утворитися даний біогенний амін? A patient with a diagnosis of 'malignant cartinoid' has a sharply increased serotonin content in the blood. From which amino acid can this biogenic amine be formed?

треоніну threonine

Триптофану Tryptophan

Метіоніну Methionine

лейцин leucine

аланін alanine

137 / 200
При електронномікроскопіческом дослідженні гіалінового хряща виявляються клітини з добре розвиненою гранулярной ендоплазматичної мережею, комплексом Гольджі. Яку функцію виконують ці клітини? Electron microscopic examination of hyaline cartilage reveals cells with a well-developed granular endoplasmic reticulum, Golgi complex. What function do these cells perform?

Депонування жиру Fat Deposit

Депонування глікогену Glycogen storage

Руйнування межкл еточного речовини хряща Destruction of cartilage intercellular substance

Освіта міжклітинної речовини Education of the intercellular substance

Трофіка хрящової тканини Cartilage tissue trophic

138 / 200
У хворого на шкірі обличчя поступово розвинулася бляшка з некрозом і виразкою в центрі. При патогистологическом дослідженні біоптату виявлено розростання атипових епітеліальних клітин з великою кількістю патологічних мітозів. Який найбільш ймовірний діагноз? The patient's facial skin gradually developed a plaque with necrosis and an ulcer in the center. Pathohistological examination of the biopsy revealed growth of atypical epithelial cells with a large number of pathological mitoses. What is the most likely diagnosis?

Папілома Papilloma

Фиброма Fibroma

Рак шкіри Skin cancer

Саркома Sarcoma

Трофическая виразка Tropical ulcer

139 / 200
У хворого міастенією після призначення прозерину з'явилася нудота, діарея, посмикування м'язів мови і скелетних м'язів. Чим можна купірувати інтоксикацію? A patient with myasthenia gravis developed nausea, diarrhea, and twitching of the tongue and skeletal muscles after being prescribed Proserin. What can be done to stop intoxication?

мезатон mesaton

ізадрину isadrin

фізостигмін physostigmine

метацин methacin

Пиридостигмина бромидом Pyridostigmine bromide

140 / 200
У хворого внутрішньосуглобове перелом шийки стегнової кістки. Спостерігається асиметрія головки. До акая структура буде найбільш ймовірно пошкоджена? The patient has an intra-articular fracture of the neck of the femur. An asymmetry of the head is observed. Which structure will most likely be damaged?

Запірательний нерв Obstructive nerve

Зв'язка головки стегнової кістки Ligament of femoral head

Стегновий нерв Femoral nerve

запирательной артерія obstructive artery

Круговий пояс Circular belt

141 / 200
У хворого інфаркт міокарда в області передньої стінки лівого шлуночка. У басейні якої артерії виникло порушення кровообігу? The patient had a myocardial infarction in the area of the front wall of the left ventricle. In the basin of which artery did the circulatory disorder occur?

Предсердно - шлуночкових гілок лівої вінцевої артерії Atrial - ventricular branches of the left coronary artery

Передньої міжшлуночкової гілки лівої вінцевої артерії Anterior interventricular branch of the left coronary artery

Лівою крайової гілки лівої вінцевої артерії Left marginal branch of the left coronary artery

Передніх шлуночкових гілок правої вінцевої артерії Anterior ventricular branches of the right coronary artery

Огинає гілки лівої вінцевої артерії Envelopes the branches of the left coronary artery

142 / 200
Хворий, що страждав хронічним гнійним остеомієлітом, помер від хронічної ниркової недостатності. При розтині виявлено великі щільні нирки беложелтого кольору з сальним блиском на зрізі. Який найбільш веро ятний діагноз? A patient suffering from chronic purulent osteomyelitis died of chronic renal failure. At autopsy, large, dense, whitish-yellow kidneys with a greasy sheen on the section were found. What is the most likely diagnosis ?

Підгострий гломерулонефрит Subacute glomerulonephritis

Хронічний гломерулонефрит Chronic glomerulonephritis

Гострий некротичний нефроз Acute necrotic nephrosis

Септичний нефрит Septic nephritis

Амилоидоз нирок Kidney amyloidosis

143 / 200
До лікаря звернулися пацієнти з подібними скаргами: слабість, болі в кишечнику, розлад шлунково-кишкового тракту. Після дослідження фекалій з'ясувалося, що термінової госпіталізації підлягає один з пацієнтів, у якого виявлено цисти з чотирма ядрами. Для якого найпростішого характерні такі цисти? Patients turned to the doctor with similar complaints: weakness, pain in the intestines, gastrointestinal tract disorders. After examining the feces, it became clear that one of the of a patient in whom cysts with four nuclei were found. What kind of protozoa is characterized by such cysts?

балантидії ballantidia

Кишкова амеба Intestinal amoeba

Тріхомонада Trichomonad

Лямблії Lamblia

Дизентерийная амеба Dysenteric amoeba

144 / 200
У хворого після важкої травми розвинувся шок і з'явилися ознаки гострої ниркової недостатності (ОПН). Що є провідним механізмом розвитку гострої ниркової недостатності в даному випадку? After severe trauma, the patient developed shock and showed signs of acute renal failure (AKI). What is the leading mechanism of development of acute renal failure in this case?

Зниження онкотичного тиску крові Reduction of blood oncotic pressure

Підвищення тиску в капсулі клубочка Increased pressure in the glomerular capsule

Падіння артеріального тиску Drop in blood pressure

Підвищення тиску в ниркових артеріях Increased pressure in renal arteries

Порушення відтоку сечі Urine outflow disorder

145 / 200
У немовляти відзначається блювота і пронос, загальна дистрофія, гепато- і спленомегалія. При припинення вигодовування молоком симптоми зменшуються. Який основний наследст венний дефект буде відзначатися в патогенезі? The infant has vomiting and diarrhea, general dystrophy, hepato- and splenomegaly. When breastfeeding is stopped, the symptoms decrease. What main hereditary defect will be noted in the pathogenesis?

Недостатність глюкозо-6-фосфатдегідрогенази Glucose-6-phosphate dehydrogenase deficiency

Порушення обміну галактози Galactose metabolism disorder

Порушення обміну тирозину Tyrosine metabolism disorder

Гиперсекреция залоз зовнішньої секреції Hypersecretion of exocrine glands

Порушення обміну фенілаланіну Phenylalanine metabolism disorder

146 / 200
З хімічного виробництва в токсикологічне відділення доставлений хворий з отруєнням ртуттю. Який препарат слід використати в даній ситуації? A patient with mercury poisoning was brought from the chemical industry to the toxicology department. What drug should be used in this situation?

ізонітрозину isonitrosine

Активоване вугілля Activated carbon

Налоксон Naloxone

Ентеросорбент СКН Enterosorbent SKN

Унітіол Unithiol

147 / 200
У хворого в 2 рази збільшена щитовидна залоза. При пальпації заліза щільна, поверхня нерівномірна, горбиста. При гістологічному дослідженні - дифузна інфільтрація тканин залози лімфоцитами, плазматичними клітинами з утворенням фолікулів і посилене розростання сполучної тканини. Який най більш ймовірний діагноз? The patient's thyroid gland is 2 times enlarged. On palpation, the iron is dense, the surface is uneven, bumpy. In histological examination - diffuse infiltration of the gland tissues by lymphocytes, plasma cells with the formation follicles and increased growth of connective tissue. What is the most likely diagnosis?

Зоб Ріделя Riedel's goiter

Дифузний токсичний зоб Diffuse toxic goiter

Зоб Хасімото Zob Hashimoto

Ендемічний зоб Endemic goiter

Спорадичний зоб Sporadic goiter

148 / 200
Дитина 10 років переніс кілька атак ревматизму. При його клінічному обстеженні було встановлено, що мали місце запальні явища в суглобах, і виявилися ознаки недостатності мітрального клапана. Яке з патологічних явищ у даного хворого можна віднести до поняття 'хвороба'? A 10-year-old child suffered several attacks of rheumatism. During his clinical examination, it was established that inflammatory phenomena occurred in the joints, and signs of mitral valve insufficiency were found. Which of the pathological can the phenomenon in this patient be attributed to the concept of 'disease'?

Недостатність мітрального клапана Mitral valve insufficiency

Ревматизм Rheumatism

Запалення суглобів Inflammation of joints

Порок мітрального клапана Mitral valve disease

Артрит Arthritis

149 / 200
У хворого діагностовано мегалобластная анемія. Недостатнє кількість якого речовини може призводити до розвитку цієї хвороби? The patient has been diagnosed with megaloblastic anemia. An insufficient amount of which substance can lead to the development of this disease?

Меди Honey

Гліцину Glycine

Магнію Magnesium

ціанокобаламін cyanocobalamin

холекальциферол cholecalciferol

150 / 200
При дослідженні крові у хворого в иявлена виражена гіпоглюкоземія натщесерце. При дослідженні біоптату печінки виявилося, що в клітинах печінки НЕ відбувається синтез глікогену. Недостатність якого ферменту є причиною захворювання? During a blood test, the patient showed severe fasting hypoglycemia. A liver biopsy showed that glycogen synthesis does NOT occur in the liver cells. What enzyme deficiency is the cause of the disease?'

Фруктозодіфосфатази Fructose diphosphatases

піруваткарбоксілази pyruvate carboxylases

фосфорилазу phosphorylase

альдолаза aldolase

глікогенсинтетазу glycogen synthetase

151 / 200
При розтині жінки 40 років, яка страждала ревматоїдним артритом, виявлено збільшену щільну селезінку. На розрізі її тканина коричнево - червоного кольору з збільшеними фолікулами, які мають вигляд напівпрозорих сірувато - білуватих зерен. Який патологічний процес найбільш вірогідний? The autopsy of a 40-year-old woman suffering from rheumatoid arthritis revealed an enlarged, dense spleen. On cross-section, her tissue is brownish-red in color with enlarged follicles that look translucent grayish - whitish grains. What pathological process is the most likely?

глазурного селезінка glaze spleen

Сагова селезінка Sago spleen

порфірний селезінка porphyry spleen

Сальна селезінка Sebaceous Spleen

Гіаліноз селезінки Hyalinosis of the spleen

152 / 200
В лабораторії досліджують мокроту хворого на туберкульоз. Який метод пофарбовані ания слід використовувати для виявлення збудників туберкульозу? The sputum of a tuberculosis patient is examined in the laboratory. What method of stained ania should be used to detect tuberculosis pathogens?

Циля - Нільсена Target - Nielsen

Грама - Синьова Grama - Sinyova

Нейссера Neissera

Гімза - Романовського Giemsa - Romanovsky

Бурри - Гінса Burrs - Ginsa

153 / 200
У новонародженого діагностовано порушення розвитку міокарда шлуночка. З порушенням розвитку якого ембріонального джерела пов'язана ця патологія? A newborn was diagnosed with a violation of the development of the myocardium of the ventricle. What embryonic source is this pathology associated with a violation of development?

мезенхіми mesenchyma

ектодерма ectoderm

ентодерми endoderms

парієтальної спланхноплеври parietal splanchnopleura

Вісцеральної спланхноплеври Visceral splanchnopleura

154 / 200
У хворого запальний гнійний процес шкіри першого межпальцевого проміжку ноги. Які лімфатичні вузли є регіональними для зазначеного ділянки і відреагують болем і припухлістю? The patient has an inflammatory purulent process of the skin of the first interdigital space of the leg. Which lymph nodes are regional for the specified area and will react with pain and swelling?

Зовнішні клубові External club

Поверхневі пахові Superficial inguinal

Задні великогомілкової і підколінні Posterior tibial and popliteal

Передні великогомілкової Tibia anterior

Поверхневі і глибокі пахові Superficial and deep inguinal

155 / 200
Аспірин надає проти вовоспалітельное дію, так як пригнічує активність циклооксигенази. Рівень будь біологічно активних речовин буде при цьому знижуватися? Aspirin has an anti-inflammatory effect, as it suppresses the activity of cyclooxygenase. Will the level of any biologically active substances decrease?

йодтироніни iodothyronine

Катехоламінів Catecholamines

біогенні амінів biogenic amines

простагландинів prostaglandins

лейкотрієнів leukotrienes

156 / 200
З гнійної рани хворого виділений патогенний стафілокок і визначена чутливість його до антибіотиків: пеніцилін - зона затримки росту 8 мм ; оксацилін - 9 мм ; ампіцилін - 10 мм ; гентаміцин - 22 мм ; линкомицин - 11 мм. Який антибіотик необхідно вибрати для лікування хворого? A pathogenic staphylococcus was isolated from the purulent wound of the patient and its sensitivity to antibiotics was determined: penicillin - growth retardation zone 8 mm; oxacillin - 9 mm; ampicillin - 10 mm; gentamicin - 22 mm; lincomycin - 11 mm. Which antibiotic should be chosen to treat the patient?

Ампіцилін Ampicillin

Линкомицин Lincomycin

Гентамицин Gentamicin

Оксациллин Oxacillin

Пеніцилін Penicillin

157 / 200
В медико - генетичну консультацію звернулася жінка з приводу ризику захворювання на гемофілію у свого сина. Її чоловік страждає цим захворюванням з народження. Жінка і її батьки здорові в відношенні гемофілії. Яка ймовірність появи хвороби у хлопчика в даній родині? A woman turned to medical and genetic counseling regarding the risk of hemophilia in her son. Her husband has suffered from this disease since birth. The woman and her parents are healthy with regard to hemophilia What is the probability that a boy in this family will have the disease?

50% хлопчиків будуть хворими 50% of boys will be sick

25% хлопчиків будуть хворими 25% of boys will be sick

Усі хлопчики будуть здоровими All boys will be healthy

Все хлопчики будуть хворими All boys will be sick

75% хлопчиків будуть хворими 75% of boys will be sick

158 / 200
Хворий 55 років спостерігається у ендокринолога з приводу порушення ендокринної функції підшлункової залози, що проявляється зменшенням кількості гормону глюкагону в крові. Функція яких клітин цієї залози порушена в цьому випадку? A 55-year-old patient is seen by an endocrinologist for a violation of the endocrine function of the pancreas, which is manifested by a decrease in the amount of the hormone glucagon in the blood. Which cells of this gland are not functioning in this case?

А-клітини острівців Лангерганса A-cells of the islets of Langerhans

S-клітини острівців Лангерганса S-cells of islets of Langerhans

бета-клітини острівців Лангерганса Beta cells of the islets of Langerhans

РР-клітини острівців Лангерганса PP cells of the islets of Langerhans

B2-клітини острівців Лангерганса B2 cells of the islets of Langerhans

159 / 200
У чоловіка 42 років, який страждає на подагру, в крові підвищена концентрація сечової кислоти. Для зниження рівня сечової кислот и йому призначений аллопуринол. Конкурентною інгібітором якого ферменту є алопуринол? A 42-year-old man suffering from gout has an increased concentration of uric acid in his blood. To reduce the level of uric acid, he is prescribed allopurinol. A competitive inhibitor of which enzyme is allopurinol ?

аденозіндезамінази adenosine deaminases

Гіпоксантінфосфорібозілтрансферази Hypoxanthine phosphoribosyltransferases

ксантиноксидазу xanthine oxidase

Аденінфосфорібозілтрансферази Adenine phosphoribosyltransferases

Гуаніндезамінази Guanindeaminases

160 / 200
У молодого людини в м'яких тканинах лівого стегна з'явилось безболісне новоутворення без чітких меж. У біоптаті тканини новоутворення нагадують риб'яче м'ясо, складаються з незрілих фібробластоподібних клітин з численними митозами, проростають в м'язи. Який найбільш ймовірний діагноз? A painless neoplasm without clear boundaries appeared in the soft tissues of the left thigh of a young man. In the biopsy, the neoplasm resembles fish flesh, consists of immature fibroblast-like cells with numerous mitoses, sprouting into muscles. What is the most likely diagnosis?

Рак Cancer

Фиброма Fibroma

Міома Myoma

міосаркома myosarcoma

Фібросаркома Fibrosarcoma

161 / 200
У хворого 24 років через півтори тижні після важкої стрептококової ангіни проявилася набряклість обличчя, підвищився артеріальний тиск. Гематурія і протеїнурія 1,2 г / л. У крові виявлено антистрептококових анти тіла і зниження компонентів комплементу. У микрососудах яких структур найбільш імовірна локалізація скупчень імунних комплексів, що зумовили розвиток нефропатії? One and a half weeks after a severe strep throat, a 24-year-old patient developed facial swelling, increased blood pressure. Hematuria and proteinuria 1.2 g/l. Anti-streptococcal antibodies were detected in the blood antibodies and a decrease in complement components. In the microvessels of which structures is the most probable localization of clusters of immune complexes that led to the development of nephropathy?

Клубочки Cupcakes

Проксимальний відділ канальців Proximal section of tubules

Спадний відділ канальців Descending tubule

Піраміди Pyramids

Петля Генле Loop of Henle

162 / 200
У чоловіка 36 років з черепно - мозкової травмою дихання слабке, пульс ниткоподібний, рефлекси відсутні. Який шлях введення пірацетаму найбільш доцільний в даному випадку? A 36-year-old man with a craniocerebral injury has weak breathing, thready pulse, no reflexes. What route of piracetam administration is most appropriate in this case?

Внутрішньовенний Intravenous

Пероральний Oral

Ректальний Rectal

Інгаляційний Inhalation

Підшкірний Subcutaneous

163 / 200
При мікроскопічному дослідженні біоптату шийки матки у хворої на хронічний ендоцервікоз виявлена клітинна і ядерна атипія багатошарового плоского епітелію, патологічні мітози, а також рогові перлини в глибині епітеліальних шарів. Який наиболе е ймовірний діагноз? During a microscopic examination of a biopsy of the cervix in a patient with chronic endocervicosis, cellular and nuclear atypia of the multilayered squamous epithelium, pathological mitoses, as well as horn pearls in the depth of the epithelial layers were found. What What is the most likely diagnosis?

Залозистий рак Glandular cancer

Плоскоклітинний рак з зроговінням Squamous cell carcinoma with keratinization

Перехідно рак Transient cancer

Плоскоклітинний рак без зроговіння Squamous cell carcinoma without keratinization

Анапластичний рак Anaplastic cancer

164 / 200
У хворого виник спазм гладенької мускулатури бронхів. Фізіологічно обгрунтованим буде dикористання для зняття нападу активаторів: The patient had a spasm of the bronchial smooth muscles. Will the use of any activators to relieve the attack be physiologically reasonable?

Β-адренорецепторів Β - adrenoblockers

α- і β-адренорецепторів α - and β - adrenoblockers

М-холінорецепторів M - cholinergic receptors

Н-холінорецепторів H - cholinergic receptors

α-адренорецепторів α - adrenoceptors

165 / 200
При дослідженні каріотипу у пацієнта були виявлені два типи клітин в однаковому кількості з хромосомними наборами 46XY і 47XXY. Який найбільш ймовірний діагноз? During the karyotype study, two types of cells were found in the patient in equal numbers with chromosome sets 46XY and 47XXY. What is the most likely diagnosis?

Синдром Патау Patau syndrome

Синдром Клайнфельтера Klinefelter syndrome

Синдром Дауна Down Syndrome

Нормальний каріотип Normal karyotype

Моносомія - Х Monosomy - X

166 / 200
При всіх формах розмноження (статеве і нестатеве розмноження) елементарними дискретними єдиний іцамі спадковості є: In all forms of reproduction (sexual and asexual reproduction), the elementary discrete elements of heredity are:

Один ген One gene

Дві ланцюжка молекули ДНК Two strands of a DNA molecule

Один нуклеотид One nucleotide

Одна ланцюжок молекули ДНК One strand of DNA molecule

Одна пара нуклеотидів One pair of nucleotides

167 / 200
В хірургічний кабінет звернувся чоловік, якого покусала невідома собака. Широкі рвані рани локалізовані на обличчі. Яку лікувально - профілактичну допомогу потрібно надати для профілактики сказу? A man who was bitten by an unknown dog came to the surgery room. Wide lacerations are localized on the face. What medical and preventive care should be provided to prevent rabies?

Госпіталізувати хворого і містити під наглядом лікаря Hospitalize the patient and keep under the supervision of a doctor

Призначити комбіновану антибіотикотерапію Prescribe combined antibiotic therapy

Терміново ввести нормальний гамма - глобулін Urgently enter normal gamma globulin

Почати імунізацію антирабічною вакциною Start immunization with rabies vaccine

Терміново ввести вакцину АКДС Urgently administer the DPT vaccine

168 / 200
При аналізі крові у хворого залишковий азот склав 48 ммоль / л, сечовина -15,3 ммоль / л. Про захворюванні якого органу свідчать результати цього аналізу? During the patient's blood analysis, residual nitrogen was 48 mmol/l, urea -15.3 mmol/l. What organ disease do the results of this analysis indicate?

Печінки Livers

Кишечника Intestine

Шлунка Stomach

Нирок Kidney

селезінки spleens

169 / 200
У жінки 62 років розвинулася катаракта (помутніння кришталика) на тлі цукрового діабету. Посилення якого процесу при діабеті є причиною помутніння кришталика? A 62-year-old woman developed cataracts (clouding of the lens) against the background of diabetes mellitus. What process intensification in diabetes is the cause of clouding of the lens?

ліполіз lipolysis

глюконеогенез gluconeogenesis

протеолізу білків protein proteolysis

Глікозіт ілірованія білків Glycosit ilirovaniya protein

кетогенез ketogenesis

170 / 200
Терапія анаприліном позитивно вплинула на динаміку хвороби у жінки 44 років, яка страждає стенокардією. Який головний механізм дії цього препарату? Anaprilin therapy had a positive effect on the dynamics of the disease in a 44-year-old woman suffering from angina pectoris. What is the main mechanism of action of this drug?

Зниження потреби і збільшення надходження кисню в міокард Decreasing the demand and increasing the supply of oxygen to the myocardium

Збільшення надходження кисню в міокард Increased oxygen supply to the myocardium

Зменшення енерговитрат міокарда внаслідок зниження навантаження Decreased energy expenditure of the myocardium due to reduced load

Зменшення окислювального обміну в міокарді внаслідок блокади ферментів циклу Кребса Reduction of oxidative metabolism in the myocardium due to blockade of enzymes of the Krebs cycle

Блокада β - адреноблокатори і зниження потреби міокарда в кисні Blockade β - adrenoblockers and reduction of myocardial oxygen demand

171 / 200
У людини внаслідок тривалого голодування швидкість клубочкової фільтрації зросла на 20%. Що є найбільш вірогідною причиною зміни фільтрації в зазначених умовах? In a person, due to prolonged starvation, the rate of glomerular filtration increased by 20%. What is the most likely reason for the change in filtration under these conditions?

Зниження онкотичного тиску плазми крові Reduction of oncotic pressure of blood plasma

Підвищення системного артеріального тиску Increased systemic blood pressure

Підвищення ниркового плазмотока Increase in renal plasma flow

Підвищення коефіцієнта фільтрації Increase the filtering factor

Вище ня проникності ниркового фільтру Increased renal filter permeability

172 / 200
В яких клітинах на протязі життя НЕ спостерігається мітоз, і кількісне зміст ДНК залишається постійним? In which cells during life, mitosis is NOT observed, and the quantitative content of DNA remains constant?

В м'язових (поперечно - смугастих) In muscular (transversally striated)

В нейронах In neurons

В м'язових (гладких) In muscular (smooth)

В кровотворних In hematopoiesis

У епідермісі In the epidermis

173 / 200
У хворої 55 років виявлено збільшення розмірів гіпофіза, гіперплазія кори надниркових залоз. АТ - 190/90 мм рт. ст.; зміст глюкози в крові - 20 ммоль / л, має місце глюкозурія, ожиріння, гірсутизм. Для якої патології характерні виявлені зміни? In a 55-year-old patient, an increase in the size of the pituitary gland, hyperplasia of the adrenal cortex was detected. Blood pressure - 190/90 mm Hg; blood glucose - 20 mmol / l, there is glucosuria, obesity, hirsutism. For what pathology are the detected changes characteristic?

Хвороба Барракера - Симмондса Barraker-Simmonds disease

Хвороба Аддісона Addison's disease

Хвороба Іценко - Кушинга Itsenko-Cushing's disease

Синдром Іценко - Кушинга Itsenko-Cushing syndrome

Адіпозогенітальная дистрофія Adiposogenital dystrophy

174 / 200
При аутопсії померлого на шкірі дистальних відділів нижніх кінцівок виявлено багряно - червоні плями, бляшки і вузли (саркома Капоши). Виявлено також гостра пневмонія, викликана пневмоцистами. Для якого захворювання характерні дані зміни? During the autopsy of the deceased, purple-red spots, plaques and nodules (Kaposh's sarcoma) were found on the skin of the distal parts of the lower limbs. Acute pneumonia caused by pneumocysts was also detected. For which What changes are characteristic of the disease?

Кір Measles

Сибірська виразка Anthrax

Грип Flu

СНІД AIDS

Дифтерія Diphtheria

175 / 200
У жінки 45 років в період цвітіння трав з'явилося гостре запальне захворювання верхніх дихальних шляхів і очей: гіперемія, набряк, слизові виділення. Який вид лейкоцитозу буде найбільш характерним при цьому? A 45-year-old woman developed an acute inflammatory disease of the upper respiratory tract and eyes during the flowering period of herbs: hyperemia, edema, mucous secretions. What type of leukocytosis will be most characteristic with?

Еозинофілія Eosinophilia

Нейтрофілія Neutrophilia

Моноцитоз Monocytosis

Базофілія Basophilia

Лимфоцитоз Lymphocytosis

176 / 200
У жінки виявлено пухлину яєчника. Показана операція. Яку зв'язку повинен перерізати хірург, щоб відділити яєчник від матки? A woman has an ovarian tumor. Surgery is indicated. What ligament should the surgeon cut to separate the ovary from the uterus?

Зв'язку, підвішуючу яєчник Ligament suspending the ovary

Власну зв'язку яєчника Ovary connection

Бічну пупкову зв'язку Lateral umbilical cord

Широку зв'язку матки Wide connection of the uterus

Круглу зв'язку матки Round ligament of the uterus

177 / 200
У хворого в анамнезі відзначений тривалий зростання кінцівок, подовжені ' павукові ' пальці, дефекти кришталика ока, аномалії серцево - судинної системи. Інтелект в нормі. Які ф енотіпіческіе ознаки ще можуть бути у цього хворого: The patient has a history of prolonged limb growth, elongated 'spider' fingers, eye lens defects, abnormalities of the cardiovascular system. Intelligence is normal. What are the phenotypic signs this patient may still have:

Порушення розвитку сполучної тканини Disruption of connective tissue development

Розщеплення м'якого та твердого піднебіння Cleft soft and hard palate

Недорозвинення нижньої щелепи Underdevelopment of the lower jaw

Недорозвинення гонад Underdevelopment of gonads

Плоске обличчя і широке перенісся Flat face and wide bridge of nose

178 / 200
У хворого з головним болем, високою температурою, ознобом, кашлем з мокротиння виділили палички овоидной форми з біполярної забарвленням, грамнегативні. В мазку з бульонной культури розташовуються ланцюжками, на агарі утворюють колонії R- форми. Для якого захворювання це характерно? In a patient with a headache, high temperature, chills, cough, ovoid-shaped rods with a bipolar color, gram-negative, were isolated from sputum. In a smear from a broth culture, they are arranged in chains, on agar forms R-shaped colonies. What disease is this characteristic of?

Дифтерія Diphtheria

Стрептококова ангіна Streptococcal sore throat

Менінгококовий назофарингіт Meningococcal nasopharyngitis

Туберкульоз Tuberculosis

Чума Plague

179 / 200
При гістологічному дослідженні стінки аорти виявлено велику кількість ксантомних клітин, розташованих переважно в інтимі. При якому захворюванні можлива така картина? During the histological examination of the aortic wall, a large number of xanthoma cells were found, located mainly in the intima. In what disease is such a picture possible?

Гіпертонічна хвороба Hypertensive disease

Сифілітичний мезаортит Syphilitic mesoorthitis

Неспецифічний аортоартеріїт Nonspecific aortoarteritis

нодозной периартериит nodose periarteritis

Атеросклероз Atherosclerosis

180 / 200
В приймально - діагностичне відділення доставлена жінка 38 років з маточним кровотечею. Які найбільш ймовірні змінений ія з боку крові відбудуться? A 38-year-old woman with uterine bleeding was brought to the reception-diagnostic department. What are the most likely changes in blood?

Збільшення гематокритного числа Hematocrit increase

Еритроцитоз Erythrocytosis

Зменшення гематокритного числа Decreasing hematocrit number

Лейкоцитоз Leukocytosis

Лейкопенія Leukopenia

181 / 200
У хворого камінь загального жовчного протока припинив надходження жовчі в кишечник. Порушення якого процесу травлення при цьому спостерігається? In the patient, a stone in the common bile duct has stopped the flow of bile into the intestines. What digestive process is disturbed?

Переваривание вуглеводів Digestion of carbohydrates

Всмоктування білків Absorption of proteins

Всмоктування вуглеводів Carbohydrate absorption

Переваривание жирів Digesting fats

Переваривание білків Digesting proteins

182 / 200
В лікарню доставили хворого на цукровий діабет в несвідомому стані. Дихання типу Кусмауля, артерії ально тиск 80/50 мм рт. ст., запах ацетону з рота. Накопиченням в організмі яких речовин можна пояснити виникнення даних розладів? A diabetic patient was brought to the hospital in an unconscious state. Kussmaul-type breathing, arterial pressure 80/50 mm Hg, the smell of acetone from the mouth. Accumulation What substances in the body can explain the occurrence of these disorders?

Модифікованих ліпопротеїдів Modified lipoproteins

Молочної кислоти Lactic acid

Складних вуглеводів Complex carbohydrates

Кетонових тел Ketone bodies

Вугільної кислоти Carbonic acid

183 / 200
В відділення травматології доставлений хворий з розчавлених м'язової тканини. Який біохімічний показник сечі при цьому буде збільшений? A patient with crushed muscle tissue was brought to the traumatology department. What biochemical indicator of urine will be increased?

Мінеральні солі Mineral salts

Креатинін Creatinine

Загальні ліпіди Total lipids

Глюкоза Glucose

Сечова кислота Uric acid

184 / 200
У хворого на туберкульоз після тривалого лікування з'явилися шум і дзвін у вухах, зниження слуху, висип на шкірі, набряк слизових оболонок і порушилася координація рухів. Після скасування препарату стан хворого значно покращився. Який препарат приймав хворий? After long-term treatment, a tuberculosis patient developed noise and ringing in the ears, decreased hearing, a rash on the skin, swelling of the mucous membranes, and impaired coordination of movements. After cancellation the patient's condition improved significantly with the drug. What drug did the patient take?

Стрептомицина сульфат Streptomycin sulfate

Рифампіцин Rifampicin

Ізоніазид Isoniazid

Етамбутол Etambutol

Бепаск Bepask

185 / 200
В лікарню швидкої допомоги доставили дитину 7 років в стані алергічного шоку, що розвинувся після укусу оси. У крові підвищена концентрація гістаміну. В результаті якої реакції утворюється цей амін? A 7-year-old child was brought to the emergency hospital in a state of allergic shock, which developed after a wasp bite. Histamine concentration is increased in the blood. As a result of which reaction is this amine formed?

Дезамінування Demining

дегидрированием by dehydrogenation

Гідроокісленія Hydrooxidation

декарбоксилюванню decarboxylation

Відновлення Recovery

186 / 200
У дитини, хворої на дифтерію, через 10 днів після введення антитоксичної протидифтерійної сироватки з'явилися висипання на шкірі, які супроводжувалися сильним свербінням, підвищилася температура тіла до 38 ° С, з'явилися болі в суглобах. Яка причина цих явищ? In a child with diphtheria, 10 days after the injection of anti-toxic anti-diphtheria serum, rashes appeared on the skin, which were accompanied by severe itching, and the body temperature rose to 38 ° C, joint pains appeared. What is the cause of these phenomena?

Анафілактична реакція Anaphylactic reaction

Сироваткова хвороба Serum sickness

Атопія Atopy

Гіперчутливість сповільненого типу Delayed type hypersensitivity

Контактна алергія Contact allergy

187 / 200
У пацієнта, який півтора місяці тому назад переніс інфаркт міокарда, діагностований синдром Дресслера з характерною тріадою: перикардит, плеврит, пневмонія. Причиною його розвитку вважається: The patient, who suffered a myocardial infarction a month and a half ago, was diagnosed with Dressler's syndrome with a characteristic triad: pericarditis, pleurisy, pneumonia. The cause of its development is considered to be:

Викид в кров міокардіальних ферментів Release of myocardial enzymes into the blood

Сенсибілізація організму антигенами міокарда Sensitization of the body by myocardial antigens

Інтоксикація організму продуктами некрозу Intoxication of the body by products of necrosis

Зниження резистентності до інфекційних агентів Decreasing resistance to infectious agents

Активація сапрофітної мікрофлори Activation of saprophytic microflora

188 / 200
Під час емоційного збудження частота серцевих скорочень у людини 30 років досягла 112 в хв. Який відділ проводить системи серця відповідає за це зміна? During emotional excitement, the heart rate of a 30-year-old man reached 112 per minute. What part of the heart system is responsible for this change?

Атріовентрикулярний вузол Atrioventricular node

Синоатріальна вузол Sinoatrial node

Пучок Гіса His Bundle

Ніжки пучка Гіса His Bundle Legs

Волокна Пуркіньє Purkinje fibers

189 / 200
Хворий госпіталізований з гострою серцево - судинної недостатністю. Який препарат необхідно використовувати в даному випадку? The patient is hospitalized with acute cardiovascular insufficiency. What drug should be used in this case?

Адреналін Adrenaline

Дигитоксин Digitoxin

Строфантин Strophantin

Дигоксин в таблетках Digoxin in tablets

Аміодарон Amiodarone

190 / 200
Гризуни є резервуаром збудників багатьох хвороб. З чим це пов'язано в першу чергу? Rodents are a reservoir of pathogens of many diseases. What is the reason for this in the first place?

Приналежність гризунів до важливим компонентам наземних біоцен озов Belonging of rodents to important components of terrestrial biocene lakes

Біологічні особливості гризунів, що сприяють обміну паразитами і збудниками з людиною Biological features of rodents that contribute to the exchange of parasites and pathogens with humans

Властивість гризунів швидко розмножуватися Property of rodents to reproduce quickly

Приналежність гризунів до найбільш численному ряду класу ссавців Rodents belong to the most numerous series of the class of mammals

Проживання в умовах, де ектопаразити використовують гризунів як джерело живлення Living in conditions where ectoparasites use rodents as a food source

191 / 200
У 4 місячного дитини яскраво виражені прояви рахіту. Розладів травлення НЕ відзначається. Дитина багато знаходиться на сонці. В протягом 2 х місяців дитина отримував вітамін D 3, однак прояви рахіту НЕ зменшилися. Порушенням синтезу якої речовини можна пояснити розвиток рахіту у цього дитини? A 4-month-old child has pronounced manifestations of rickets. Digestive disorders are NOT noted. The child is in the sun a lot. For 2 months, the child received vitamin D 3, but the manifestations rickets did NOT decrease. Violation of the synthesis of which substance can explain the development of rickets in this child?

Паратгормону Parathyroid hormone

Тироксину Thyroxine

кальцитоніну calcitonin

кальцитріолом calcitriol

Інсуліну Insulin

192 / 200
Тварині, сенсибілізованій туберкуліном, внутрибрюшинно введений туберкулін. Через 24 години при лапаратомии виявлено венозну гіперемію і набряк очеревини. В мазках - відбитках з очеревини велика кількість лімфоцитів і моноцитів. Який патологічний процес у тварини? In an animal sensitized by tuberculin, tuberculin was administered intraperitoneally. After 24 hours, at laparotomy, venous hyperemia and edema of the peritoneum were found. In smears - prints from the peritoneum, a large number of lymphocytes and monocytes. What is the pathological process in the animal?

Гнійне запалення Purulent inflammation

Алергічне запалення Allergic inflammation

Серозное запалення Serous inflammation

Асептичне запалення Aseptic inflammation

Фібринозне запалення Fibrinous inflammation

193 / 200
Альбіноси погано переносять сонячне загар, у них з'являються опіки. Порушення метаболізму який кислоти лежить в основі цього явища? Albinos do not tolerate sunburn well, they get burns. Which acid metabolism disorder is the basis of this phenomenon?

Триптофану Tryptophan

глутамінова кислоти glutamic acid

Гістидину Histidine

фенілаланін phenylalanine

Метіоніну Methionine

194 / 200
При повному (з водою) аліментарному голодуванні розвинулись генералізовані набряки. Який з патогенетичних факторів у цьому випадку є провідним? With complete (with water) alimentary starvation, generalized edema developed. Which of the pathogenetic factors in this case is the leading one?

Підвищення онкотичного тиску тканинної рідини Increased oncotic pressure of tissue fluid

Зниження осмотичного тиску плазми крові Decrease in blood plasma osmotic pressure

Зниження гідростатичного тиску міжклітинної рідини Reduction of intercellular fluid hydrostatic pressure

Підвищення осмотичного тиску міжклітинної рідини Increased osmotic pressure of intercellular fluid

Зниження онкотичного тиску плазми крові Reduction of oncotic pressure of blood plasma

195 / 200
При штовханні штанги спортсмен закидає голову назад для максимального підвищення тонус а м'язів - розгиначів верхніх кінцівок. Де розташовані центри рефлексів, що виникають при цьому? When pushing the barbell, the athlete throws his head back to maximally increase the tone and extensor muscles of the upper limbs. Where are the centers of reflexes that occur during this?

В червоних ядрах In red kernels

В базальних гангліях In the basal ganglia

В ядрах Дейтерса In Deiters cores

У рухової корі In the motor cortex

У спинному мозку In the spinal cord

196 / 200
Хворому проводять правобічну пульмонектомію по приводу раку легкого. Який порядок розміщення анатомічних утворень кореня правого легкого (в напрямку зверху вниз)? The patient is undergoing a right-sided pulmonectomy for lung cancer. What is the order of anatomical formations of the root of the right lung (from top to bottom)?

Бронх, артерія, вени Bronchu, artery, veins

Відня, артерія, бронх Vienna, artery, bronchus

Артерія, вени, бронх Artery, veins, bronchus

Артерія, бронх, вени Artery, bronchus, veins

197 / 200
При реєстрації ЕКГ хворого з гіперфункцією щитовидної залози зареєстровано збільшення частоти серцевих скорочень. Скорочення якого елемента ЕКГ про це свідчить? When recording the ECG of a patient with thyroid hyperfunction, an increase in heart rate was recorded. The reduction of which ECG element indicates this?

Сегмента PQ PQ segment

Інтервалу PQ PQ Interval

Комплексу QRS QRS complex

Інтервалу RR RR Interval

Інтервалу Р-Т Interval P-T

198 / 200
При розтині труп а хворого, який помер від легеневої недостатності, виявлено збільшене легке з вогнищами темно - червоного, білого, рожево - жовтого кольору, некротичний трахеобронхіт. Яким захворюванням найбільш ймовірно страждав хворий? At the autopsy of a patient who died of pulmonary insufficiency, an enlarged lung with foci of dark red, white, pinkish-yellow color, necrotic tracheobronchitis, was found. What disease was the patient most likely to suffer?

Фиброзирующий альвеоліт Fibrosing alveolitis

Крупозная пневмонія Croupous pneumonia

Корова пневмонія Cow pneumonia

Токсична форма грипу Toxic form of flu

Грипозна пневмонія Influenza pneumonia

199 / 200
При обстеженні у хворого виявлено підвищений вміст ліпопротеїнів низької щільності в сироватці крові. Яке захворювання можна очікувати у цього хворого? During the examination, the patient has an increased content of low-density lipoproteins in the blood serum. What disease can be expected in this patient?

Атеросклероз Atherosclerosis

Гастрит Gastritis

Пошкодження нирок Kidney damage

Запалення легенів Pneumonia

Гострий панкреатит Acute pancreatitis

200 / 200
Внаслідок блокади іонних каналів мембрани клітини її потенціал спокою зменшився з -90 мВ до -70 мВ. Які канали заблоковані? Due to the blockade of ion channels of the cell membrane, its resting potential decreased from -90 mV to -70 mV. Which channels are blocked?

Хлорні Chlorni

Натрієві Sodium

Калієві Potassium

Кальцієві Calcium

Магнієві Magnesium